فهرست مطالب

نشریه خانواده درمانی کاربردی
سال سوم شماره 4 (پیاپی 13، زمستان 1401)

  • تاریخ انتشار: 1401/12/08
  • تعداد عناوین: 34
|
  • شهرزاد شفائی، بهرام میرزاییان*، رمضان حسن زاده صفحات 1-17

    هدف:

     هدف این پژوهش تعیین اثربخشی توانبخشی شناختی بر عملکرد اجرایی و عملکرد حافظه فعال در سالمندان مبتلا به افسردگی غیر بالینی بود.

    روش پژوهش: 

    پژوهش حاضر یک تحقیق کاربردی و از نظر روش تحقیق از نوع تحقیقات نیمه آزمایشی از نوع تحقیقات پیش آزمون- پس آزمون و پیگیری همراه با گروه کنترل بود. جامعه ی آماری این پژوهش شامل کلیه سالمندان مبتلا به افسردگی غیر بالینی که تعداد آنها 90 نفر زن و مرد بین دی ماه 1399 تا خرداد سال 1400 به مرکز مشاوره مهرگان در طول شش ماه مراجعه نمودند. نمونه مورد مطالعه 45 نفر سالمندان بود که به صورت نمونه گیری هدفمند انتخاب و به صورت تصادفی به دو گروه مساوی مداخله و یک گروه کنترل تقسیم شدند (هر گروه 15 نفر). دوره پیگیری پس از گذشت سه ماه از اتمام پس آزمون انجام شد. داده در این پژوهش با استفاده از پرسشنامه افسردگی بکII (بک، استیر و براون، 1996) پرسشنامه ی عملکرد اجرایی، پرسشنامه عملکرد حافظه فعال: توسط والات ازووی، پرادات دیل و ازووی (2012) بدست آمد. در این پژوهش از آزمون اندازه گیری مکرر و نرم افزارهای SPSS.22 استفاده شد.

    یافته ها

    نتایج نشان داد که توانبخشی شناختی بر عملکرد اجرایی و عملکرد حافظه فعال در سالمندان مبتلا به افسردگی غیر بالینی موثر بود و این نتایج تا دوره پیگیری ماندگار بود.

    نتیجه گیری

    می توان نتیجه گرفت که توانبخشی شناختی روش موثری برای بهبود عملکرد اجرایی و عملکرد حافظه فعال در سالمندان مبتلا به افسردگی غیر بالینی است و می توان از آن در جهت کاهش مشکلات سالمندان استفاده کرد.

    کلیدواژگان: توانبخشی، عملکرد اجرایی، حافظه، سالمندان، افسردگی
  • زهرا رضایی روزبهانی، آزیتا چهری*، گلاویژ محمودی صفحات 18-32
    هدف

    این تحقیق با هدف اثربخشی درمان مبتنی بر پذیرش و تعهد بر مشکلات خواب و حل مساله انجام شد.

    روش پژوهش: 

    این پژوهش از نوع نیمه تجربی با طرح پیش ‏آزمون- پس ‏آزمون با گروه کنترل و پیگیری 3 ماهه بود. جامعه آماری این پژوهش شامل تمامی دانش آموزان دختر 14 تا 16 ساله مشغول به تحصیل در مقطع متوسطه شهر کرمانشاه در سال 400-1399 بود. با روش نمونه ‏گیری در دسترس تعداد 60 دانش ‏آموز دختر انتخاب و به شیوه تصادفی ‏سازی (قرعه‏ کشی) به دو گروه مساوی 20 نفری گمارش شدند. گردآوری داده ‏ها با پرسشنامه حل مساله (هپنر و پترسون، 1988) و پرسشنامه سنجش خواب (بویس و همکاران، 1989) انجام شد. برای تحلیل داده ‏ها از نرم افزار SPSS نسخه 25 با روش‏های آمار توصیفی، پیش فرض ‏های آماری و تحلیل واریانس با اندازه‏ گیری مکرر و آزمون بن‏فرونی استفاده شد.

    یافته ها

    نتایج نشان داد که درمان پذیرش و تعهد به طور معناداری بر بهبود مهارت حل مساله و ابعاد آن در افراد گروه آزمایش اثربخش بوده است. همچنین، نتایج نشان داد که درمان پذیرش و تعهد بر مهارت حل مساله افراد گروه آزمایش تاثیر مثبتی داشته است.

    نتیجه گیری

    بر اساس یافته های پژوهش حاضر می توان چنین نتیجه گرفت که درمان مبتنی بر پذیرش و تعهد با بهره گیری از فنونی همانند کاربرد تکنیک های ذهنی، مشاهده خود به عنوان زمینه، تکنیک های گسلش شناختی، تصریح ارزش ها و عمل متعهدانه می تواند به عنوان یک درمان کارآمد جهت کاهش مشکلات خواب و بهبود شیوه های حل مسیله در دختران نوجوان مورد استفاده قرار گیرد.

    کلیدواژگان: درمان پذیرش و تعهد، مشکلات خواب، حل مساله، نوجوان
  • فرحناز امیرلو، هایده صابری*، میترا حکیم شوشتری صفحات 33-53
    هدف

    ترکیب ذهن آگاهی، خودکارآمدی والدینی و فراوالدگری در کاهش تنیدگی والدین به ویژه مادران نقش بسزایی داشته و مدیریت تنیدگی والدینی جهت ارتقای بهزیستی امری مهم محسوب می‎شود.

    روش پژوهش: 

    پژوهش حاضر با هدف بررسی نقش میانجیگر ذهن آگاهی و خودکارآمدی والدینی در تبیین روابط علی بین فراوالدگری و تنیدگی والدینی در مادران کودکان مبتلا به اوتیسم انجام شد. طرح پژوهش همبستگی و مبتنی بر روش مدل یابی معادلات ساختاری و جامعه آماری شامل همه مادران دارای کودک 2 تا 14 سال مبتلا به اختلال اوتیسم مراجعه کننده به کلینیک های درمانی شهر تهران در سال 1400 بودند که 250 نفر به روش نمونه گیری هدفمند انتخاب و به پرسشنامه های تنیدگی والدگری آبیدین (1995)، پرسشنامه ذهن آگاهی (براون و رایان، 2003)، پرسشنامه فراوالدگری (هاوک و هولدن، 2006) و خودکارآمدی والدینی (دومکا و همکاران، 1996) پاسخ دادند. تجزیه و تحلیل داده ها با استفاده از نرم افزارAMOS  نسخه 24 انجام شد.

    یافته ها

    نتایج نشان داد که ضریب مسیر کل بین فراوالدگری و تنیدگی والدینی (01/0>p، 456/0-=β) منفی و ضریب مسیر بین ذهن آگاهی و تنیدگی والدینی (01/0>p، 474/0-=β) و خودکارآمدی والدینی و تنیدگی والدینی (01/0>p، 386/0-=β) منفی و در سطح 01/0 معنادار بودند که خودکارآمدی والدینی و ذهن آگاهی رابطه بین فرا والدگری و تنیدگی والدگری را به صورت منفی و معناداری میانجیگری کرد.

    نتیجه گیری

    فراوالدگری به صورت مستقیم و ذهن آگاهی و خودکارآمدی والدینی به صورت مستقیم و غیرمستقیم، تنیدگی والدینی را در مادران کودکان مبتلا به اوتیسم پیش بینی می کند. ضرورت دارد جهت ارایه مداخلات به منظور کاهش یا جلوگیری از سطوح بالای تنیدگی والدینی در خانواده‎ها به مداخلات مبتنی بر ذهن آگاهی، خودکارآمدی و فراوالدگری در روابط والد فرزندی توجه ویژه‎ای گردد.

    کلیدواژگان: تنیدگی والدینی، ذهن آگاهی، فرا والدگری، خودکارآمدی والدینی، اوتیسم
  • نگار روستا، مجید قدمی*، حسین کشاورز افشار، قدسی احقر صفحات 54-69

    هدف :

    مقایسه اثربخشی مشاوره گروهی با رویکرد گشتالت درمانی و واقعیت درمانی بر تنظیم هیجان دانش آموزان دوره اول متوسطه شهر تهران بود.

    روش پژوهش: 

    پژوهش حاضر از نوع نیمه آزمایشی با طرح پیش آزمون-پس آزمون با گروه گواه و مرحله پیگیری بود. جامعه آماری شامل کلیه دختران نوجوان دوره اول متوسطه مدارس دولتی شهر تهران در سال تحصیلی 99-1398 بود. 45 نفر با استفاده از روش نمونه گیری هدفمند انتخاب و به صورت تصادفی در سه گروه 15 نفری قرارگرفتند. گروه آزمایش اول 10 جلسه 90 دقیقه ای مشاوره گروهی با رویکرد گشتالت درمانی و گروه آزمایش دوم 10 جلسه 90 دقیقه ای مشاوره واقعیت درمانی را دریافت کردند. ابزار پژوهش پرسشنامه تنظیم هیجان گارنفسکی و کاریج (2006) بود. داده ها با استفاده از روش تحلیل واریانس تجزیه و تحلیل شدند.

    یافته ها

    بین گروه آزمایش اول مشاوره گروهی با رویکرد گشتالت درمانی، گروه آزمایش دوم واقعیت درمانی و گروه کنترل در تنظیم هیجان تفاوت معنی دار وجود دارد اما بین رویکرد گشتالت درمانی و واقعیت درمانی در تنظیم هیجان تفاوت معناداری مشاهده نشد (001/0˃p).

    نتیجه گیری

    مشاوره گروهی گشتالت درمانی و واقعیت درمانی بر تنظیم هیجان تاثیر مثبت و معناداری داشته اند. بنابراین مشاوره گروهی گشتالت درمانی روشی و واقعیت درمانی روش های مناسبی برای افزایش تنظیم هیجان دانش آموزان هستند. با توجه به نتایج به دست آمده پیشنهاد می شود که مشاوران مدارس و مراکز روان درمانی از روش های مشاوره گروهی با رویکرد گشتالت درمانی و واقعیت درمانی در بهبود تنظیم هیجان استفاده کنند.

    کلیدواژگان: گشتالت درمانی، واقعبت درمانی، تنظیم هیجان، دختران نوجوان
  • سیده فاطمه بهاری سقالکساری، محمدکاظم فخری*، شعبان حیدری صفحات 70-89
    هدف

    این پژوهش با هدف تعیین اثربخشی روان درمانی مثبت نگر بر رضایت از تصویر تن و شادکامی در دختران نوجوان ناراضی از تصویر تن بود.

    روش پژوهش:

     پژوهش حاضر از نوع نیمه آزمایشی با طرح پیش آزمون- پس آزمون و پیگیری بود. از بین دختران نوجوان ناراضی از تصویر تن نمونه ای به حجم 30 نفر به روش نمونه گیری غیرتصادفی هدفمند انتخاب و در دو گروه آزمایش (روان درمانی مثبت نگر) و گواه به صورت تصادفی ساده جایگزین شدند. (15 نفر در هر گروه). پس از ارزیابی، 30 نفر از دختران نوجوان ناراضی از تصویر تن که نمره پایین تر نمره برش پرسشنامه رضایت از تصویرتن را کسب نمودند و شرایط لازم را داشتند، شناسایی و سپس در دو گروه 15 نفری، یک گروه آزمایشی و یک گروه گواه اختصاص یافتند. سپس 8 جلسه 90 دقیقه ای روان درمانی مثبت نگر سلیگمن (2006) در گروه آزمایش به صورت حضوری و با رعایت پروتکل های بهداشتی انجام شد. داده ها با استفاده از پرسشنامه شادکامی (آرگایل و همکاران، 1989) و مقیاس رضایت از تصویرتن (سویوتو و گارسیا، 2002) جمع آوری شد. تجزیه و تحلیل داده ها با استفاده از آزمون آماری تحلیل واریانس با اندازه گیری مکرر با نرم افزار SPSS-22 انجام گرفت.

    یافته ها: 

    نتایج نشان داد که در مراحل پس آزمون و پیگیری، روان درمانی مثبت نگر بر رضایت از تصویر تن (86/49=F، 001/0>P) و شادکامی (53/13=F، 001/0>P) اثربخش بودند و در مراحل پس آزمون و پیگیری، درمان مبتنی بر پذیرش و تعهد بر رضایت از تصویر تن و شادکامی اثربخش بودند و در سایر موارد تفاوت معنی داری مشاهده نشد.

    نتیجه گیری

    به طور کلی با توجه به نتایج پژوهش، احتمالا بتوان از روان درمانی مثبت نگر و درمان مبتنی بر پذیرش و تعهد در بهبود مسایل روان شناختی دختران نوجوان استفاده نمود.

    کلیدواژگان: روان درمانی مثبت نگر، شادکامی، دختران نوجوان، تصویر تن
  • زهرا اخگری، حسین ابراهیمی مقدم*، رحیم داوری صفحات 90-115

    هدف:

     هدف از این پژوهش بررسی ارایه الگوی ساختاری پیش بینی تحمل پریشانی بر اساس صفات تاریک شخصیت و حمایت اجتماعی ادراک شده با میانجی گری سبک های دفاعی در مراجعین خواستار طلاق بود.

    روش پژوهش: 

    روش این مطالعه تحلیلی -همبستگی به روش مدل سازی معادلات ساختاری بود. جامعه آماری پژوهش حاضر مراجعین خواستار طلاق در کلینیک های مشاوره خانواده شهر تهران در سال 1400 بودند. حجم نمونه 260 نفر به شیوه نمونه ‏گیری در دسترس انتخاب و به مقیاس تحمل پریشانی سیمونز و گاهر (2005)، مثلث تاریک شخصیت جانسون و وبستر (2010)، مقیاس چندبعدی حمایت اجتماعی ادراک شده زیمت و همکاران (1988) و پرسشنامه مکانیزم دفاعی اندروز و همکاران (1993) پاسخ دادند. سپس الگوی پیشنهادی از طریق مدل سازی معادلات ساختاری با استفاده از SPSS و AMOS نسخه 24 تحلیل شد.

    یافته ها

    نتایج نشان داد مسیرهای مستقیم صفات شخصیت و سبک های دفاعی بر روی تحمل پریشانی معنادار بود (01/0>P). مسیرهای مستقیم صفات شخصیت بر روی حمایت اجتماعی ادراک شده و سبک‏های دفاعی (رشد یافته و رشد نایافته) بر روی تحمل پریشانی معنادار بود (01/0>P). همچنین مسیرهای مستقیم حمایت اجتماعی ادراک شده بر روی سبک های دفاعی(روان آزرده و رشد نایافته) معنادار بود (01/0>P). مسیر مستقیم حمایت اجتماعی ادراک شده بر روی تحمل پریشانی و ضریب صفات شخصیت بر روی سبک دفاعی روان آزرده و ضریب مسیر مستقیم حمایت اجتماعی ادراک شده بر روی سبک دفاعی رشد یافته معنادار نبود (05/0<P). همچنین نتایج بیانگر آن بود که اثرات غیرمستقیم حمایت اجتماعی ادراک شده از طریق نقش میانجی سبک های دفاعی بر تحمل پریشانی معنادار بود (077/0=RMSEA، 912/0=GFI، 959/0=IFI و 957/0=CFI)، اما مسیر غیرمستقیم صفات شخصیت بر روی تحمل پریشانی با نقش میانجی سبک های دفاعی معنادار نبود.

    نتیجه گیری

    این پژوهش، مدل ساختاری پژوهش اصلاح شده پژوهش برازش مطلوب و قابل قبول با داده های پژوهشی داشتو این گام مهمی در جهت شناخت عوامل موثر بر تحمل پریشانی در مراجعینی که خواستار طلاق هستند.

    کلیدواژگان: تحمل پریشانی، صفات تاریک شخصیت، حمایت اجتماعی ادراک شده، سبک های دفاعی، طلاق
  • زهرا باقری ماسی مرده، مجید ضرغام حاجبی*، عباس حبیب زاده صفحات 116-134

    هدف :

    هدف این پژوهش تعیین برازش معادلات ساختاری شکست عاطفی براساس سردرگمی هویت با میانجی گری مکانیزم های دفاعی بود.

    روش پژوهش: 

    روش پژوهش حاضر توصیفی و طرح پژوهش همبستگی از نوع مدل معادلات ساختاری بود. جامعه آماری این مطالعه شامل کلیه دختران نوجوان دانش آموز دارای شکست عاطفی در سال تحصیلی 99-98 شهرستان نورآباد بود. نمونه این پژوهش شامل 250 نفر که با استفاده از روش نمونه گیری هدفمند انتخاب شدند. داده ها با استفاده از پرسشنامه سندرم ضربه عشقی (راس، 1999)، پرسشنامه سازمان شخصیت (کرنبرگ، 2002) و پرسشنامه مکانیزم های دفاعی (اندروز و همکاران، 1993) جمع آوری شد. تحلیل داده های پژوهش با استفاده از ضریب همبستگی پیرسون، مدل معادلات ساختاری و نرم افزارهایSPSS نسخه 22 و AMOS نسخه 22 صورت گرفت. سطح معناداری در این پژوهش 0٫05 در نظر گرفته شد.

    یافته ها

    نتایج نشان داد که سردرگمی اثر غیرمستقیم بر شکست عاطفی داشته است، بنابراین وجود رابطه غیرمستقیم بین متغیرهای پژوهش با 95 درصد اطمینان تایید می شود (05/0>P).

    نتیجه گیری

    می توان نتیجه گرفت که سازمان شخصیت دارای نقش میانجی در رابطه بین مکانیزم های دفاعی و شکست عاطفی داشته است.

    کلیدواژگان: سازمان شخصیت، سردرگمی هویت، شکست عاطفی، مکانیزم های دفاعی
  • نازنین حبیبی زنجانی، سعیده برازیان*، حسن احدی صفحات 135-149

    هدف:

     تعیین اثربخشی رفتار درمانی عقلانی هیجانی بر تصویر بدنی، شاخص توده بدنی و کاهش اضطراب اجتماعی نوجوانان دارای اضافه وزن بود.

    روش پژوهش: 

    روش پژوهش حاضر از نوع نیمه تجربی با طرح پیش آزمون و پس آزمون و پیگیری با گروه گواه بود. جامعه آماری این پژوهش را نوجوانان دختر و پسر تشکیل می دادند که با مشکل اضافه وزن و چاقی به کلینیک های شهر تهران در سال 1398 مراجعه کرده بودند که از بین آنها 40 نفر با روش نمونه گیری در دسترس به عنوان نمونه انتخاب شدند. پس از انتخاب افراد به طور تصادفی در دو گروه آزمایش و گروه گواه (هر گروه 20 نفر) گمارش شدند. مداخله گروه آزمایش به صورت گروه درمانی و براساس رفتاردرمانی عقلانی هیجانی (الیس و درایدن، 1997) و شامل هشت جلسه به مدت 120 دقیقه بود. داده ها با استفاده از پرسشنامه های نگرانی درباره تصویر بدن لیتلتون و همکاران (2005)، و پرسشنامه اضطراب اجتماعی کانور و همکاران (2000) و روش تحلیل واریانس با اندازه گیری مکرر و نرم افزار spss.22 تحلیل شد. سطح معناداری در این پژوهش، 05/0 در نظر گرفته شد.

    یافته ها

    نتایج نشان داد که رفتار درمانی عقلانی-هیجانی در تصویر بدنی (29/25=F، 001/0>P)، شاخص توده بدنی (59/3=F، 036/0=P)، و کاهش اضطراب اجتماعی (49/5=F، 026/0=P) نوجوانان دارای اضافه وزن اثربخش بودند (001/0>P).

    نتیجه گیری

    براساس یافته های پژوهش می توان گفت رفتار درمانی عقلانی-هیجانی می توانند تاثیرات مثبتی را بر روی تصویر بدنی، شاخص توده بدنی و کاهش اضطراب اجتماعی نوجوانان داشت.

    کلیدواژگان: رفتار درمانی عقلانی هیجانی، تصویر بدنی، شاخص توده بدنی، اضطراب اجتماعی
  • علی رجبی وندچالی، رمضان حسن زاده*، قدرت الله عباسی صفحات 150-168

    هدف:

     پژوهش حاضر با هدف تعیین اثربخشی درمان شناختی رفتاری بر افکار خود آسیب رسانی و بهزیستی روانشناختی در دانش آموزان دچار انحلال روابط عاشقانه بود.

    روش پژوهش: 

    این پژوهش نیمه آزمایشی با طرح پیش آزمون و پس آزمون و پیگیری با دو گروه آزمایش و گواه بود. جامعه آماری شامل 587 نفر از مراجعه کنندگان به مرکز مشاوره شهر ساری بود که از بین آنها 60 نفر از دانش آموزان که دارای انحلال روابط عاشقانه و بر اساس معیارهای ورود و خروج به مطالعه انتخاب شدند و به طور تصادفی در دو گروه آزمایش گواه هر گروه 30 نفر قرار گرفتند. شرکت کننده ها، پرسشنامه خود آسیب رسانی سانسون و سانسون (2010) و پرسشنامه بهزیستی روانشناختی ریف (1989) را در سه نوبت پیش آزمون، پس آزمون و پیگیری تکمیل کردند. گروه آزمایش 8 جلسه درمان شناختی رفتاری دریافت کردند و گروه گواه هیچ مداخله ای دریافت نکرد. داده ها با استفاده از روش تحلیل اندازه گیری مکرر و با نرم افزار spss.22 تحلیل شد.

    یافته ها

    در گروه آزمایش، میانگین (انحراف معیار) افکار خود آسیب رسان از 40/19 (84/1) در پیش آزمون به 80/9 (56/0) در پس آزمون کاهش یافت ولی در گروه گواه میانگین (انحراف معیار) افکار خود آسیب رسان در پیش آزمون 93/18 (90/1) و در پس آزمون 80/18(61/1) بود که تفاوتی نداشت . همچنین در گروه آزمایش میانگین (انحراف معیار) بهزیستی روانشناختی در پیش آزمون برابر با 73/36 (60/11) بود که در پس آزمون به 80/99 (12/4) افزایش یافت ولی در گروه گواه میانگین (انحراف معیار) بهزیستی روانشناختی در پیش آزمون 53/37 (13/14) و در مرحله پس آزمون 46/37 (91/13) بدست آمد.

    نتیجه گیری

    مطالعه نشان داد که درمان شناختی رفتاری منجر به کاهش افکار خود آسیب رسان و افکار خودکشی گرایانه و موجب افزایش و بهزیستی روانشناختی در دانش آموزان دچار انحلال روابط عاشقانه شد.

    کلیدواژگان: درمان شناختی رفتاری افکار خود آسیب رسان، بهزیستی روانشناختی، انحلال روابط عاشقانه
  • وحیده السادات فاطمی، عبدالله شفیع آبادی*، جواد خلعتبری، عبدالحسن فرهنگی صفحات 169-184
    هدف

    هدف از پژوهش حاضر، تعیین اثربخشی درمان پذیرش و تعهد بر مهارت‏های ارتباطی دانش آموزان دختر دوره دوم متوسطه بود.

    روش

    روش پژوهش نیمه آزمایشی با طرح پیش آزمون، پس آزمون با گروه گواه همراه با مرحله پیگیری 2 ماهه بود. در این پژوهش جامعه آماری دانش‏آموزان دختر دوره دوم متوسطه شهر تهران در سال تحصیلی 99-1398 بودند که با استفاده از روش نمونه گیری تصادفی ساده تعداد 30 نفر از آنان در دو گروه آزمایش و گواه قرار گرفتند (هر گروه 15 نفر). گروه آزمایش تحت 12 جلسه 90 دقیقه ای جلسات درمانی پذیرش و تعهد اقتباس از پترسون، ایفرت، فینگولد و دیویدسون (2009) قرار گرفت؛ اما گروه گواه هیچ گونه مداخله‏ای دریافت نکردند و در لیست انتظار باقی ماندند. از پرسشنامه تجدیدنظر شده مهارت‏های ارتباطی کویین‏دام (2004) به منظور گردآوری اطلاعات استفاده شد. تجزیه و تحلیل اطلاعات به دست آمده از اجرای پرسشنامه ها از طریق نرم افزار SPSS نسخه 24 در دو بخش توصیفی و استنباطی (تحلیل واریانس با اندازه گیری مکرر و آزمون تعقیبی بن فرونی) انجام پذیرفت.

    یافته ها:

     نتایج نشان داد درمان پذیرش و تعهد در مرحله پسآزمون و پیگیری نسبت به گروه گواه اثربخشی معناداری بر درک پیام (37/6=F، 004/0=P)، تنظیم هیجان (62/8=F، 001/0>P)، گوش دادن (57/4=F، 017/0=P)، بینش به ارتباط (04/16=F، 001/0>P)، ابراز وجود (16/9=F، 001/0>P) دختران نوجوان دارد.

    نتیجه گیری: 

    بر اساس نتایج پژوهش حاضر، می توان گفت که درمان پذیرش و تعهد می -تواند به عنوان شیوه‏های درمانی برای بهبود مهارت‏های ارتباطی دختران نوجوان در محیط های آموزشی و درمانی به کار برده شود.

    کلیدواژگان: درمان پذیرش و تعهد، مهارت‏های ارتباطی، دختران
  • عفت شیرازی، مهدی قاسمی مطلق*، بهرنگ اسماعیلی شاد، ابوالفضل بخشی پور صفحات 185-207
    هدف

    هدف پژوهش حاضر، مدل یابی نشانه های شخصیت مرزی بر اساس دلبستگی: نقش میانجی تمایز یافتگی خود، و بدتنظیمی هیجانی بود.

    روش پژوهش:

     پژوهش حاضر از نوع توصیفی و به روش همبستگی (با استفاده از مدل یابی معادلات ساختاری) است. جامعه آماری مورد مطالعه در این پژوهش شامل کلیه دختر و پسر جوان دانشجوی دانشگاه آزاد شهر تهران بود که در سال تحصیلی 98-99 مشغول به تحصیل بودند. نمونه پژوهش حاضر شامل 370 نفر بود که به روش نمونه گیری هدفمند انتخاب شدند. داده ها با استفاده از پرسشنامه شخصیت مرزی (لیشنرنیگ، 1999)، پرسشنامه تجدید نظر شده در روابط نزدیک - ساختار رابطه (فرالی، والرو برنان، 2000)، پرسشنامه تمایزیافتگی خود اسکورن و فریدلندر (1998) و مقیاس دشواری تنظیم هیجانی گرتز و رومر (2004) به دست آمد. در پژوهش حاضر میانگین، انحراف معیار، همبستگی و نرمال بودن توزیع متغیر های پژوهش با استفاده از آمار توصیفی بررسی شد. همچنین به منظور تحلیل داده ها و پاسخ به پرسش های پژوهش از روش مدل یابی معادلات ساختاری استفاده می شود که بعد از تحلیل عاملی تاییدی - در قسمت مدل اندازه‎گیری در قسمت مدل معادله ساختاری روابط علی موجود میان متغیرهای مکنون مورد بررسی قرار می گیرد. در قسمت مدل تابع ساختاری شدت روابط علی (مستقیم، غیر مستقیم و کل) میان متغیرهای مکنون و مقدار واریانس تبیین شده در کل مدل مشخص می شود.

    یافته ها: 

    نتایج نشان داد دلبستگی (41/0=β)، تمایزیافتگی خود (24/0=β)، و تنظیم هیجان (31/0=β) دارای اثر مستقیم بر نشانه های شخصیت مرزی است (001/0>P). تمایز یافتگی خود و بدتنظیمی هیجانی در رابطه نشانه های شخصیت مرزی و دلبستگی دارای نقش میانجی بود.

    نتیجه گیری

    در نتیجه می توان نتیجه گرفت مدل یابی نشانه های شخصیت مرزی بر اساس دلبستگی با نقش میانجی تمایز یافتگی خود، و بدتنظیمی هیجانی از برازش مطلوب برخوردار است.

    کلیدواژگان: شخصیت مرزی، دلبستگی، تمایز یافتگی خود، بدتنظیمی هیجانی
  • سیده سارا میرزائیان گیزه رود، نادر منیرپور*، محمداسماعیل اکبری، مجید ضرغام حاجبی صفحات 208-216
    ‏هدف

    بررسی مدل ساختاری کیفیت زندگی زنان مبتلا به سرطان پستان براساس شخصیت تیپ D با نقش میانجی متابعت درمانی می باشد.‏

    روش پژوهش: 

    در این مطالعه کاربردی اطلاعات توصیفی زنان مبتلا به سرطان پستان مراجعه کننده به بیمارستان شهدای تجریش و کلینیک آذر تهران در سال 1399-1398 جمع آوری گردید. حجم نمونه بر 440 نفر تعیین گردید . ابزار گردآوری داده ها شامل پرسشنامه جمعیت شناختی کیفیت زندگی (EORTC QLQ -BR23) ،مقیاس تیپ شخصیتیD و پرسشنامه متابعت کلی و اختصاصی بیماران مزمن توسط هیز و همکاران (1994) بود.

    یافته ها

    ضریب مسیر متغیر شخصیت تیپ Dبر متابعت درمانی زنان مبتلا به سرطان پستان که به مقدار 694/0 و همچنین آماره t به مقدار 121/7 است، ‏ضریب مسیر متابعت درمانی بیماری بر کیفیت بیماران مورد مطالعه که به مقدار 887/0 و همچنین آماره t به مقدار 283/9 و نهایتا ضریب مسیر متغیر شخصیت تیپ Dبر کیفیت زندگی زنان مبتلا به سرطان پستان به مقدار 523/0 و همچنین آماره t به مقدار 120/4 مشاهده گردید که همگی تاثیر معناداری داشت.

    نتیجه گیری:

     شخصیت تیپ D با میانجیگری متابعت درمانی بر کیفیت زندگی زنان مبتلا به سرطان پستان تاثیر معناداری داردوباعث کاهش کیفیت زندگی می شود.

    کلیدواژگان: کیفیت زندگی، شخصیت تیپ D، متابعت درمانیت، سرطان پستان
  • عادل امیرخانلو، یارعلی دوستی*، رضا دنیوی صفحات 217-242

    هدف :

    این پژوهش به منظور مقایسه اثر بخشی درمان شناختی- رفتاری با آموزش هوش هیجانی بر سازگاری اجتماعی نوجوانان مبتلا به اختلال سلوک انجام شد.

    روش پژوهش:

     روش پژوهش حاضر، نیمه آزمایشی با طرح پیش آزمون، پس آزمون و پیگیری با گروه گواه (دو گروه آزمایش و یک گروه گواه) بود. جامعه آماری پژوهش را کلیه نوجوانان مبتلا به اختلال سلوک که درکانون اصلاح تربیت تهران در سال 1398حضور دارند، تشکیل دادند. نمونه ازبین افراد آزمودنی‎های مبتلا به اختلال سلوک غربال شدند، 45 نفر از نوجوانان به صورت تصادفی در سه گروه (دو گروه آزمایش و یک گروه گواه هر گروه 15 نفر) جای گرفتند. نمونه گیری، در مرحله انتخاب آزمودنی‎ها به صورت هدفمند و در مرحله جایگزینی در گروه‎های آزمایش و گواه به صورت تصادفی انجام گرفت. گروه آزمایش اول در طی هشت جلسه براساس پروتکل شناختی- رفتاری اسمیت (2000) و گروه آمایش دوم طی هشت جلسه براساس پروتکل آموزش هوش هیجانی برادبری و گریوزف (2001) تحت آموزش قرار گرفتند اما گروه گواه در لیست انتظار قرار گرفت. ابزار گردآوری داده ها در این تحقیق پرسشنامه سازگاری اجتماعی کالیفرنیا (1953) بود. جهت تحلیل داده ها از آزمون تحلیل واریانس با اندازه گیری مکرر استفاده شد.

    یافته ها

    نتایج حاصل از تحقیق نشان داد که درمان شناختی- رفتاری و آموزش هوش هیجانی بر افزایش سازگاری اجتماعی (49/6=F، 003/0=P)، قالب های اجتماعی (74/57=F، 001/0>P)، مهارتهای اجتماعی (33/7=F، 002/0=P)، علایق ضد اجتماعی (73/7=F، 001/0>P)، روابط خانوادگی (34/4=F، 010/0=P)، روابط مدرسه ای (13/4=F، 045/0=P) و روابط اجتماعی (97/4=F، 018/0=P) نوجوانان مبتلا به اختلال سلوک اثربخش است. همچنین مقایسه دو گروه آزمایش نشان داد که درمان شناختی-رفتاری تاثیر بیشتری در افزایش سازگاری اجتماعی نوجوانان دارد (001/0>P).

    نتیجه گیری

    می توان نتیجه گرفت که درمان شناختی- رفتاری و آموزش هوش هیجانی بر افزایش سازگاری اجتماعی نوجوانان مبتلا به اختلال سلوک موثر بود.

    کلیدواژگان: درمان شناختی- رفتاری، آموزش هوش هیجانی، سازگاری اجتماعی، اختلال سلوک
  • سیما کامرانی فر، زهرا افتخار ساعدی*، فرح نادری، سحر صفرزاده صفحات 243-260

    هدف :

    هدف پژوهش حاضر مقایسه اثربخشی آموزش ایمن سازی در برابر استرس و آموزش تنظیم هیجان بر کفایت اجتماعی و رفتارهای پرخطر در دانش آموزان دختر بود.

    روش پژوهش: 

    پژوهش حاضر، از نوع پژوهش های نیمه آزمایشی با پیش آزمون و پس آزمون و پیگیری همراه با گروه گواه بود. جامعه آماری این پژوهش شامل تمامی دانش آموزان دختر دوره دوم متوسطه شهرستان رامهرمز در سال تحصیلی 99-1398 بود که در مدارس این شهر مشغول به تحصیل بودند. از این جامعه آماری، به روش نمونه گیری تصادفی چندمرحله ای 3 مدرسه انتخاب شد. دو مدرسه به طور تصادفی به گروه های آزمایشی 1 و 2 و یک مدرسه به عنوان گروه گواه گمارده شدند. تعداد گروه آموزش ایمن سازی در برابر استرس براساس پروتکل مایکنبام (2007) شامل 23 نفر و گروه آموزش تنظیم هیجان گراس (2002) شامل 22 نفر و گروه گواه شامل 25 نفر، افراد شرکت کننده در این پژوهش بودند. ابزار مورد استفاده در این پژوهش شامل پرسشنامه کفایت اجتماعی فلنر و همکاران (1990)، پرسشنامه خطرپذیری نوجوانان ایرانی زاده محمدی و احمدآبادی (1387) بود. تجزیه و تحلیل داده ها با استفاده از تحلیل واریانس با اندازه گیری مکرر به کمک نرم افزار SPSS.22 صورت گرفت

    یافته ها

    نتایج نشان داد که بین گروه های آزمایشی و گروه گواه از لحاظ متغیرهای کفایت اجتماعی (64/34=F، 001/0>P) و رفتارهای پرخطر (56/56=F، 001/0>P) تفاوت معنی داری وجود دارد. همچنین، مشخص شد که روش آموزش تنظیم هیجان تاثیر بیشتری بر میزان بهبود کفایت اجتماعی و رفتارهای پرخطر دارد.

    نتیجه گیری

    می توان نتیجه گرفت آموزش ایمن سازی در برابر استرس و آموزش تنظیم هیجان بر کفایت اجتماعی و رفتارهای پرخطر در دانش آموزان دختر موثر بود.

    کلیدواژگان: کفایت اجتماعی، رفتارهای پرخطر، استرس، تنظیم هیجان
  • آویسا خبیری پویا، محسن محمدی*، ابوالفضل کرمی، اسحق رحیمیان بوگر صفحات 261-283
    هدف

    هدف اصلی از انجام این پژوهش تدوین الگو رضایت جنسی بر اساس فاکتورهای شناختی، اجتماعی، اخلاقی است و روشن شدن اهمیت این فاکتورها در بهبود و ارتقا کیفیت رضایت رابطه جنسی سالم تر افراد می باشد.

    روش پژوهش: 

    این پژوهش از نظر هدف کاربردی و به لحاظ روش توصیفی، معادلات ساختاری می باشد. جامعه آماری این پژوهش شامل 300 نفر (129مرد و 161 زن) متاهل و مجرد که تجربه رابطه جنسی را داشته اند و از تحصیلات دیپلم تا دکتری را شامل می شدند بود. داده ها با استفاده از پرسشنامه رضایت جنسی، پرسشنامه دشواری های تنظیم، پرسشنامه رابطه جنسی، پرسشنامه مقیاس تمایز یافتگی، پرسشنامه اعتماد در روابط بین فردی و پرسشنامه عدالت اخلاقی به دست آمد. کلیه اطلاعات بدست آمده از این پژوهش به وسیله نسخه 22 نرم افزار SPSS براساس شاخص های آمار توصیفی مانند فراوانی، میانگین و انحراف معیار تجزیه و تحلیل گردید و روش تحلیل مسیر استفاده شد.

    یافته ها

    نتایج به دست آمده نشان داد رضایت جنسی با رابطه جنسی اشتراکی با توجه به مقدار 427/0با دشواری تنظیم هیجانی به مقدار 258/0، با تمایز یافتگی به مقدار 243/0 ، اعتماد در رابطه بین فردی به مقدار 269/0، عدالت اخلاقی به مقدار 175/0 با رضایت جنسی دیده می شود.

    نتیجه گیری

    ابعاد فردی و بین فردی انسان ها روی رضایت جنسی می تواند تاثیرگذار باشد. پیشنهاد می شود آموزش های لازم با توجه به ابعاد مختلف انسانی به شکل الگویی که تمرکز آن روی مسایل شناختی، روابط بین فردی و بعد اخلاقی است صورت گیرد و شرایطی برای ارتقا رضایت جنسی و تداوم آن در محیطی امن و سالم مهیا شود تا به تداوم این مسیر کمک کند.

    کلیدواژگان: رضایت جنسی، رابطه جنسی اشتراکی، دشواری تنظیم هیجانی، تمایز یافتگی، عدالت اخلاقی
  • سروش شاه بیک، آمنه معاضدیان*، پرویز صباحی، ارسلان خان محمدی اطاقسرا صفحات 284-308
    هدف

    پژوهش حاضر پیش بینی افسردگی بر اساس طرحواره های ناسازگار اولیه، تاب آوری و حمایت اجتماعی ادراک شده با در نظر گرفتن نقش میانجی شفقت بر خود و امیدواری در مردان و زنان مبتلا به سرطان بود.

    روش پژوهش:

    روش پژوهش توصیفی و از نوع همبستگی بود. جامعه ی آماری در این پژوهش شامل کلیه بیماران مبتلا به سرطان مراجعه کننده به انستیتو کانسر بیمارستان های امام خمینی و میلاد تهران بودند. تعداد افراد نمونه 300 بیمار مبتلا به سرطان بود که براساس ملاک های ورود و خروج پژوهش و با روش نمونه گیری هدفمند انتخاب شدند. ابزار پژوهش شامل پرسشنامه ی شفقت بر خود نف (2003)، پرسشنامه طرحواره های ناسازگار اولیه یانگ (1998)، پرسشنامه تاب آوری کونور-دیویدسون (2005)، پرسشنامه حمایت اجتماعی ادراک شده زیمت و همکاران (1988)، مقیاس افسردگی بک (1961) و پرسشنامه امیدواری اسنایدر (1991) بودند. داده های گردآوری شده با روش مدل سازی معادلات ساختاری تحلیل شدند.

    یافته ها:

    یافته ها نشان داد که مدل پیش بینی افسردگی بر اساس طرحواره های ناسازگار اولیه، تاب آوری و حمایت اجتماعی ادراک شده با در نظر گرفتن نقش میانجی شفقت بر خود و امیدواری براساس داده های تجربی از برازش مطلوبی برخوردار است. همچنین اثر مستقیم و غیرمستقیم طرحواره های ناسازگار اولیه، تاب آوری و حمایت اجتماعی ادراک شده با در نظر گرفتن نقش میانجی شفقت بر خود و امیدواری بر افسردگی بیماران مبتلا به سرطان مورد تایید قرارگرفت (p<0.05). شفقت بر خود به طور مستقیم بر افسردگی بیماران مبتلا به سرطان اثر می گذارد (p<0.05).همچنین امیدواری به طور مستقیم بر افسردگی بیماران مبتلا به سرطان اثر می گذارد (p<0.05).

    نتیجه گیری

    براساس نتایج پژوهش برای اینکه بیماران مبتلا به سرطان عملکرد مناسبی داشته باشند لازم است از یک سو طرحواره های ناسازگار اولیه تعدیل شده ای داشته باشند، از سوی دیگر دوره های درمانی آنها باید مبتنی بر بهبود تاب آوری و حمایت اجتماعی ادراک شده باشد.

    کلیدواژگان: افسردگی، تاب آوری، حمایت اجتماعی ادراک شده، شفقت بر خود، امیدواری
  • زهره مرادی*، فروغ جعفری صفحات 309-325

    هدف :

    هدف پژوهش حاضر بررسی نقش واسطه ای پرخاشگری ارتباطی پنهان در رابطه بین عدم تحمل بلاتکلیفی با روابط فرازناشویی بود.

    روش پژوهش:

     این پژوهش توصیفی از نوع پژوهش های همبستگی است. جامعه آماری پژوهش را زوج های مراجعه کننده برای مشاوره زناشویی به مراکز مشاوره استان البرز در سال 1398 تشکیل می دادند. نمونه ی پژوهش به صورت نمونه گیری در دسترس انتخاب شد. حجم نمونه مورد نیاز در این پژوهش بر اساس روش کوکران 260 نفر به دست آمد که با احتساب ریزش نمونه ها 254 نمونه (شامل 127 مرد و 127 زن) انتخاب شدند. ابزار پژوهش شامل مقیاس نگرش به روابط فرازناشویی ویتلی (2008)، مقیاس عدم تحمل بلاتکلیفی فریستون و همکاران (1994) و مقیاس پرخاشگری ارتباطی پنهان نلسون و کارول (2006) بودند. در این پژوهش جهت تحلیل داده ها از روش تحلیل مسیر با رعایت پیش فرض های مربوط به آن استفاده شد.

    یافته ها

    نتایج تحلیل داده ها نشان داد شاخص های برازش مدل در وضعیت مطلوبی قرار داشتند. عدم تحمل بلاتکلیفی به صورت مستقیم و غیرمستقیم (با در نظر گرفتن نقش واسطه ی پرخاشگری ارتباطی پنهان) بر روابط فرازناشویی اثر می گذارد (p<0.05). همچنین پرخاشگری پنهان بر روابط فرازناشویی به صورت مستقیم اثر می گذارد (p<0.05).

    نتیجه گیری

    براساس یافته های پژوهش می توان گفت اصلاح عدم تحمل بلاتکلیفی و پرخاشگری پنهان به عنوان پیشایندهای روابط فرازناشویی، باعث می شود کیفیت زندگی زناشویی زوجین بهبود یابد، و تاثیر معنادار در پیش گیری از بروز روابط فرازناشویی دارد.

    کلیدواژگان: پرخاشگری ارتباطی پنهان، عدم تحمل بلاتکلیفی، روابط فرازناشویی
  • ملیحه افخمی اردکانی، سعید وزیری*، یاسر رضاپور میرصالح، محمدحسین فلاح صفحات 326-346

    هدف:

     هدف این پژوهش ارایه الگوی بومی(اسلامی-ایرانی) فرزندپروری شفقت ورزانه بر اساس نظر متخصصان بود.

    روش پژوهش: 

    این پژوهش از نوع آمیخته با هدف اعتباریابی الگوی بومی اجرا می شود. جامعه آماری پژوهش حاضر در بخش کیفی و کمی کلیه متخصصان در حوزه مطالعات فرزندپروری و الگوهای بومی-اسلامی خواهند بود. نمونه ها در بخش کیفی از بین افراد متخصص به روش هدفمند و بر مبنای اشباع نظری خبرگان پژوهش و در بخش کمی به شیوه غیر تصادفی در دسترس انتخاب خواهد شد، ابزار پژوهش در بخش کیفی شامل مصاحبه عمیق و در بخش کمی نیز پرسشنامه محقق ساخته بوده است.

    یافته ها

    در نهایت این نتیجه حاصل شده است که الگوی فرزندپروری بومی اسلامی ایرانی شفقت ورزانه با مولفه های خدامحوری رفتار والدین، احوالات مادر در ایام بارداری و شیردهی، دعوت فرزندان به فضایل اخلاقی، دلسوزی عمیق برای فرزندان، دعای خیر در حق والدین و فرزندان، اعتقاد به باارزش بودن فرزندان، آموزش صحیح فرزندان، تربیت صحیح و متفاوت در هفت سال اول و دوم و سوم، الگو بودن رفتار والدین، رسیدن به من حقیقی، اهل عمل بودن از روایی کمی صوری و روایی محتوایی مطلوبی برخوردار است. یعنی ملاحظه می گردد که تمامی بارهای عاملی سیوالات بالاتر از 5/0 بدست آمده است و این حاکی از تایید روایی سازه گویه ها می باشد.

    نتیجه گیری

    طی تحلیل عاملی تاییدی نیز مشخص گردید مدل معادلات ساختاری در پژوهش حاضر از برازش مطلوبی برخوردار است. بنابراین این الگو از اعتبار کافی برخوردار است.

    کلیدواژگان: الگوی بومی، فرزندپروری، شفقت ورزی، والدین
  • محمدرضا مهدی قلی، فریده دوکانه ای فرد*، پانته آ جهانگیر صفحات 347-365
    هدف

    پژوهش حاضر با هدف تعیین برازش مدل پیش بینی دلزدگی زناشویی براساس تمایزیافتگی، هوش هیجانی و سلامت روان با میانجی گری تاب آوری در زوجین انجام شد.

    روش پژوهش:

     روش پژوهش حاضر همبستگی از نوع مدل یابی معادلات ساختاری بود. جامعه آماری شامل تمامی زوجین مراجعه کننده به مراکز مشاوره خانواده منطقه 1 شهر تهران در بازه زمانی نیمه دوم تیر تا پایان شهریور ماه سال 1400 بود، که 248 زوج (124 مرد و 124 زن) با روش نمونه گیری غیرتصادفی بصورت در دسترس انتخاب و با روش اجرای اینترنتی با استفاده از مقیاس های دلزدگی زناشویی (پاینز، 1996)، هوش هیجانی شات، و همکاران (1998)، تاب آوری کانر و دیویدسون (2003) و پرسشنامه های تمایزیافتگی خود اسکورن و دندی، (2003) و سلامت عمومی گلدبرگ و هیلر(1972) مورد ارزیابی قرار گرفتند.

    یافته ها:

     نتایج به دست آمده از ارزیابی مدل پیشنهادی به روش معادلات ساختاری نشان داد شاخص های برازش مدل از سطح مطلوبی برخوردار است. بین تمایزیافتگی (05/0>P) و سلامت روانی (01/0>P) با دلزدگی زناشویی رابطه معناداری وجود دارد، اما بین هوش هیجانی با دلزدگی زناشویی رابطه معناداری وجود ندارد (05/0<P). همچنین تاب آوری در رابطه بین تمایزیافتگی، هوش هیجانی و سلامت روان با دلزدگی زناشویی زوجین نقش میانجی ندارد.

    نتیجه گیری

    بر اساس یافته های پژوهش حاضر می توان گفت که تمایزیافتگی و سلامت روانی نقش مهمی در دلزدگی زناشویی و به طور کلی روابط زناشویی زوجین دارد، لذا توجه به متغیرهای مذکور در پیشگیری و طراحی درمان های مناسب تر به پژوهشگران و درمانگران حوزه زوج و خانواده یاری می رساند.

    کلیدواژگان: تاب آوری، تمایزیافتگی، دلزدگی زناشویی، سلامت روان، هوش هیجانی
  • رویا دیده ور، الهام ضرقامی*، مرتضی عندلیب کورایم صفحات 366-387

    هدف :

    پژوهش حاضر به دلیل نبود نسخه فارسی ابزار حاضر به بررسی ویژگی‏های روانسنجی و بررسی ساختارعاملی پرسشنامه طرحواره مثبت یانگ در دانشجویان دختر دانشگاه آزاد اسلامی واحد ورامین پرداخت. 

    روش پژوهش:

     تحقیق حاضر، از نوع تحقیق های توصیفی- کاربردی و با روش میدانی بود. ابزارهای جمع‏آوری داده‏ شامل پرسشنامه طرحواره مثبت یانگ لوییس و همکاران (2018)، فرم کوتاه پرسشنامه‏ ی طرحواره ‏ی یانگ ویلبورن و همکاران (2002)، مقیاس افسردگی و اضطراب و استرس لاویبند (1995) و پرسشنامه فرزندپروری یانگ یانگ و همکاران (2003) بود. جهت بررسی روایی صوری و محتوایی نظر 11 متخصص در حوزه روانشناسی و جهت بررسی روایی سازه و پایایی درونی 468 نمونه، جهت روایی واگرا داده ‏های 135 نمونه و به منظور ارزیابی پایایی باز-آزمایی داده ‏های 100 نمونه مورد تحلیل قرار گرفت. در این تحقیق از روش روایی محتوایی (CVR) و شاخص روایی محتوایی (CVI) برای محاسبه روایی محتوا استفاده شد. به منظور ارزیابی پایایی درونی پرسشنامه از روش آلفای کرونباخ استفاده و به منظور ارزیابی پایایی زمانی از روش باز-آزمایی، روایی واگرا از همبستگی پیرسون با نرم افزار SPSS استفاده شد. جهت بررسی روایی سازه از تحلیل عاملی تاییدی با نرم افزار LISREL استفاده شد.

    یافته ها

    نتایج پژوهش نشان داد مدل 76 سوالی با 17 عامل مورد تایید است. ضریب همبستگی پیرسون بین نمرات حاصل از دو اجرا در محدوده 68/0 تا 87/0 و همگی معنادار بودند (001/0>P) که نشان از همبستگی بالای دو آزمون و در نتیجه پایایی بازآزمایی مناسب تمامی خرده مقیاس‎های پرسشنامه طرحواره مثبت یانگ است.

    نتیجه گیری

    بنابراین می توان نتیجه گرفت مدل 76 سوالی با 17 عامل پرسشنامه طرحواره مثبت یانگ ابزاری مطلوب برای شناسایی و ارزیابی طرحواره‏های مثبت است. مطلوب است تحقیقات آینده به بررسی اعتبار وپایایی پرسشنامه در خرده فرهنگ های دیگر و طیف های سنی و شغلی مختلف بپردازند.

    کلیدواژگان: طرحواره، طرحواره‏ها‏ی ناسازگار اولیه، روایی، پایایی
  • نسرین رهبری غازانی، نادر حاجلو*، سیف الله آقاجانی صفحات 388-403
    هدف

    هدف پژوهش حاضر بررسی اثربخشی توانمندسازی خانواده مبتنی بر یادگیری اجتماعی-هیجانی بر تغییر جو عاطفی خانواده و پیشگیری از اعتیاد در نوجوانان دارای والدین وابسته به مواد بود. مواد و

    روش

    روش پژوهش حاضر نیمه آزمایشی با طرح پیش آزمون-پس آزمون و گروه کنترل و پیگیری بود که جامعه آماری آن را نوجوانان دارای والدین وابسته به مواد که جهت ترک اعتیاد به مراکز ترک اعتیاد دارویی شهر تبریز مراجعه کرده بودند تشکیل داده بود که از بین آنها 30 نفر به روش هدفمند و در دسترس نمونه گیری شده و با گمارش تصادفی در دو گروه آزمایش و کنترل گمارده شدند. داده های پژوهش با استفاده از پرسشنامه جو عاطفی خانواده نوردگاه فرد (1373) و مقیاس گرایش به اعتیاد موسوی و همکاران (1387) گردآوری و با تحلیل واریانس با اندازه گیری مکرر تجزیه و تحلیل شد.

    نتایج

    نتایج پژوهش حاضر نشان داد که توانمندسازی خانواده بر تغییر جو عاطفی خانواده و پیشگیری از اعتیاد در نوجوانان دارای والدین وابسته به مواد اثربخش بوده و این اثربخشی در پیگیری سه ماهه پایدار است.

    بحث و نتیجه گیری: 

    با توجه به نتایج پژوهش حاضر می توان نتیجه گرفت که توانمندسازی خانواده مبتنی بر یادگیری اجتماعی-هیجانی جزو درمان های اثربخش بر جو عاطفی خانواده و پیشگیری از اعتیاد است.

    کلیدواژگان: یادگیری اجتماعی-هیجانی، جو عاطفی خانواده، پیشگیری، اعتیاد
  • طیبه انسان دوست، علی اکبر ثمری*، محمدحسین بیاضی، علیرضا رجایی صفحات 404-420

    هدف :

    درد به عنوان یک حس عمومی، دارای پایه های زیستی است، و نقش عوامل روانشناختی دارای اهمیت است؛ بنابراین هدف مطالعه حاضر تعیین اثربخشی درمان پذیرش و تعهد بر بهزیستی روانشناختی بیماران مبتلابه درد مزمن عضلانی - اسکلتی بود.

    روش پژوهش: 

    روش پژوهش حاضر، نیمه آزمایشی با طرح پیش آزمون- پس آزمون و پیگیری با گروه گواه بود. جامعه آماری پژوهش شامل تمامی افراد مبتلابه درد مزمن عضلانی - اسکلتی در تابستان 1399 شهر تربت جام بودند که از بین آن ها 30 نفر از افراد واجد شرایط به صورت نمونه گیری در دسترس انتخاب شدند و به طور تصادفی در دو گروه آزمایش و گواه (هر گروه 15 نفر) قرار گرفتند. و به پرسش نامه بهزیستی روانشناختی پاسخ دادند. درمان پذیرش و تعهد طی هشت جلسه به صورت هفتگی آموزش داده شد. داده های آماری با استفاده از روش تحلیل واریانس با اندازه گیری مکرر و نرم افزار -16 SPSS در سطح معنی داری 05/0= α تحلیل شدند.

    یافته ها

    نتایج نشان دادند که درمان پذیرش و تعهد بر ارتقای بهزیستی روانشناختی (03/39=F، 001/0=P) و ابعاد آن در مرحله پس ازمون موثر بود؛ این تاثیر در محله پیگیری پایدار بود (05/0>P).

    نتیجه گیری

    بر اساس یافته های این پژوهش می توان گفت درمان پذیرش و تعهد بر افزایش بهزیستی روانشناختی بیماران مبتلابه درد مزمن عضلانی - اسکلتی موثر بود و می توان از این روش در جهت بهبود مشکلات روان شناختی بیماران مبتلابه درد مزمن عضلانی - اسکلتی استفاده نمود.

    کلیدواژگان: پذیرش و تعهد، بهزیستی روانشناختی، عضلانی - اسکلتی
  • افسانه خواجوند خوشلی* صفحات 421-436
    هدف

    هدف این پژوهش بررسی اثربخشی آموزش فرزندپروری مبتنی بر پذیرش و تعهد بر سبک های والدگری و اضطراب در مادران دارای فرزند با اختلال نارسایی توجه/ فزون کنشی بود.

    روش

    طرح پژوهش نیمه آزمایشی از نوع پیش آزمون - پس آزمون با دو گروه آزمایشی و گواه بوده است. جامعه آماری این پژوهش کلیه مادران دارای فرزند با اختلال نارسایی توجه / فزون کنشی بود. نمونه این پژوهش شامل 30 نفر از مادران دارای فرزند با اختلال نارسایی توجه/ فزون کنشی که به روش نمونه گیری در دسترس انتخاب شدند. سپس مادران گروه آزمایش تحت 9 جلسه 90 دقیقه ای هر هفته یک جلسه آموزش فرزندپروری مبتنی بر پذیرش و تعهد قرار گرفتند ولی گروه گواه هیچ مداخله ای دریافت نکردند. ابزار پژوهش پرسشنامه های اضطراب بک و پرسشنامه سبک والدگری بامریند بوده است. داده ها با استفاده از روش تحلیل واریانس با اندازه گیری مکرر و نرم افزار Spss.23 تحلیل شد.

    یافته ها: 

    نتایج نشان داد که آموزش فرزندپروری مبتنی بر پذیرش و تعهد بر سبک های والدگری مقتدرانه (001/0>P)، مستبدانه (001/0>P)، سهل گیرانه (001/0>P) و اضطراب (001/0>P) در مادران دارای فرزند با اختلال نارسایی توجه/ فزون کنشی تاثیر معناداری داشته است.

    نتیجه گیری:

     می توان نتیجه گرفت آموزش فرزندپروری مبتنی بر پذیرش و تعهد بر بر سبک های والدگری و اضطراب در مادران دارای فرزند با اختلال نارسایی توجه/ فزون کنشی موثر بود و می توان از نتایج این پژوهش در جهت کاهش مشکلات مادران دارای فرزند با اختلال نارسایی توجه/فزون کنشی استفاده کرد.

    کلیدواژگان: درمان پذیرش و تعهد، سبک های والدگری، اضطراب، نارسایی توجه، فزون کنشی
  • سید محسن ابراهیمی، رجبعلی محمدزاده ادملایی*، جمال صادقی، علی اصغر عباسی اسفجی صفحات 437-463

    هدف پژوهش حاضر بررسی نقش میانجی ویژگی های شخصیتی در رابطه بین صمیمیت خانواده مبدا با اهمال کاری تحصیلی است. بدین منظور 300 نفر از دانشجویان دختر و پسر دانشگاه آزاد بابل به صورت در دسترس پرسشنامه های خانواده مبدا (FOS)، اهمال کاری تحصیلی سولومون و راثبلوم و سنجش صفات پنج گانه شخصیتی (NEO-FFI) را تکمیل کردند. داده ها با استفاده از روش تحلیل مسیر از طریق آزمون بوت استرپ و مدل یابی معادلات ساختاری تحلیل شدند. براساس یافته های پژوهش همه ویژگی های شخصیتی روان آزرده خویی، برون گرایی، تجربه پذیری، توافق جویی و وظیفه شناسی توانستند در رابطه با صمیمیت خانواده مبدا و اهمال کاری تحصیلی به عنوان متغیر میانجی ایفای نقش کنند اما ویژگی روان آزرده خویی تاثیرش به طور منفی بود. در واقع می توان گفت در خانواده سالم رشد صمیمت و اعتماد به نفس منجر به شکل گیری ویژگی های شخصیتی همچون فعال بودن، وظیفه شناسی و نیز همدلی خواهد شد که بر پیشرفت تحصیلی و کاهش اهمال کاری تاثیرگذارند. در حالی که افراد روان آزرده خو با حس ناامنی و اضطراب بیشتر، معمولا با درگیری و تعارض، مانع ایجاد محیطی سرشار از شادی شده و کاهش حساسیت افراد نسبت به هم، کاهش اعتماد بین آنها را نیز سبب می شود. همچنین بجای بیان احساسات به طور منطقی، با رفتار پرخاشگرانه و خصومت بیشتر، مانع پیشرفت تحصیلی و افزایش اهمال کاری می شوند.

    کلیدواژگان: صمیمیت خانواده مبدا، ویوگی های شخصیتی، اهمال کاری تحصیلی
  • حسین سلیمانپور مقدم، علی سعیدی*، حسین مهدیان صفحات 464-478

    پژوهش حاضر با هدف مقایسه اثربخشی شفقت ورزی و آموزش مبتنی بر پذیرش و تعهد (اکت) بر بهزیستی روانشناختی انجام شد. و از جایی که از جمله دغدغه های متخصصان آموزشی هر جامعه ای، بررسی مشکلات عاطفی، روانی و رفتاری دانش آموزان از طرفی و ارتقاء سطح بهداشت روانی آنان از طرفی دیگر می باشد، به نظر می رسد این دو روش می توانند در این راستا به افزایش بهزیستی روانشناختی دانش آموزان کمک نمایند. این مطالعه، یک پژوهش تجربی با طرح پیش آزمون - پس آزمون باپیگیری و گروه کنترل بود. جامعه پژوهش، دانش آموزان پسر پایه یازدهم در ناحیه چهار آموزش و پرورش شهر مشهد در سال تحصیلی 1401 - 1400 بودند که با روش نمونه گیری خوشه ای، از مدارسی که دارای حداقل چهار پایه یازدهم بودند یک مدرسه به شیوه تصادفی انتخاب و سپس سه کلاس به شیوه تصادفی انتخاب و به صورت قرعه کشی در سه گروه جایگزین شدند. گروه های آزمایش در 8 جلسه 90 دقیقه ای به تفکیک با روش های شفقت ورزی و آموزش مبتنی بر پذیرش و تعهد (اکت) آموزش دیدند و گروه کنترل،آموزشی دریافت نکرد. ابزار پژوهش، مقیاس بهزیستی روانشناختی ریف (فتحی آشتیانی و همکاران، 1395) بود. داده ها در نرم افزار spss25 با روش های تحلیل واریانس اندازه گیری های مکرر عاملی تک متغیره و آزمون تعقیبی بنفرونی تحلیل شدند. نتایج حاکی از اثربخشی هر دو روش شفقت و آموزش مبتنی بر پذیرش و تعهد (اکت) در افزایش بهزیستی روانشناختی بود (001/0 <P). همچنین بین دو روش آموزشی در افزایش بهزیستی روانشناختی تفاوت معنی داری وجود نداشت (05/0 <P). با توجه به یافته های پژوهش حاضر، برای افزایش بهزیستی روانشناختی دانش آموزان می توان از روش های شفقت ورزی و آموزش مبتنی بر پذیرش و تعهد (اکت) استفاده نمود.

    کلیدواژگان: شفقت ورزی، آموزش مبتنی بر پذیرش و تعهد، بهزیستی روانشناختی
  • زهره صادقی عمروآبادی، مهناز استکی*، کامبیز پوشنه، مهدیه صالحی صفحات 479-492
    هدف

    پژوهش حاضر با هدف مقایسه میزان اثربخشی روش تراپلی بر اختلال دلبستگی واکنشی کودکان 8 تا 12 سال صورت گرفت.

    روش

    روش پژوهش، از نوع نیمه آزمایشی با طرح پیش آزمون، پس آزمون و پیگیری بود. جامعه آماری شامل کلیه کودکان 8 تا 12 سال شهر تهران که در فراخوان از طریق فضای مجازی ابراز تمایل کرده بودند، بود. از میان جامعه مذکور 32 کودک، انتخاب و در دو گروه آزمایش و کنترل قرار گرفتند. مداخله به صورت روش تراپلی به صورت هفتگی ارایه شد. شرکت کنندگان قبل از آغاز و پس از اتمام جلسات پرسشنامه اختلال دلبستگی راندولف، سیاهه رفتاری اخنباخ و آزمون دسته بندی کارت های ویسکانسین را تکمیل و سه ماه بعد پس آزمون تکرار گردید. داده ها از طریق نرم افزار SPSS با استفاده از روش آماری آنوای دو عاملی آمیخته مورد تجزیه و تحلیل قرار گرفتند.

    یافته ها: 

    نتایج حاصل از تحلیل داده ها اثرگذاری معنادار روش تراپلی را بر علایم اختلال دلبستگی واکنشی (رفتارهای ضد اجتماعی، تکانشگری و زورگویی) تایید کرد (05/0>P).

    نتیجه گیری: 

    بنابر نتایج به دست آمده در این پژوهش استفاده از رویکرد تراپلی می تواند به عنوان درمان های موثر در کاهش علایم اختلال دلبستگی واکنشی متناسب با شرایط کودک به کار گرفته شود.

    کلیدواژگان: روش تراپلی، اختلال دلبستگی واکنشی، کودکان
  • حسینعلی قربانی امیر، امید مرادی*، مختار عارفی، حمزه احمدیان صفحات 493-508
    هدف

    پژوهش حاضر با هدف مقایسه اثربخشی مولفه های شناختی رفتاری با مولفه های مبتنی بر پذیرش و تعهد بر تاب آوری و خودکنترلی زنان مطلقه بوده است.

    روش

    پژوهش حاضر از نیمه آزمایشی از نوع پیش آزمون و پس آزمون و پیگیری با گروه گواه بود که با روش نمونه گیری در دسترس، از بین کلیه زنان مطلقه مراجعه کننده به مرکز مشاوره دادگستری شهرستان بابل نیمه دوم سال 1396 و نیمه اول سال 1397، تعداد 45 نفر انتخاب و به طور تصادفی در دو گروه آزمایش و گواه تقسیم شدند. ابزار اندازه گیری شامل مقیاس تاب آوری و پرسشنامه خودکنترلی بود. برای اجرا، در ابتدا، از هر گروه پیش آزمون گرفته شد. سپس آموزش شناختی رفتاری طی 12 جلسه 90 دقیقه ای در گروه آزمایش صورت گرفت و بعد از هر دو گروه پس آزمون گرفته شد. سه ماه بعد از اجرای پس آزمون، دوره پیگیری اجرا شد. تحلیل داده ها با استفاده از تحلیل واریانس با اندازه گیری مکرر و نرم افزار Spss.22 صورت گرفت.

    یافته ها: 

    این مطالعه نشان داد که مولفه های شناختی رفتاری و مولفه های مبتنی بر پذیرش و تعهد بر تاب آوری (001/0<p) و خودکنترلی (001/0<p) در زنان مطلقه موثر بودند. این مطالعه نشان داد که درمان مبتنی بر پذیرش و تعهد نسبت به درمان شناختی رفتاری تاثیر بیشتری بر افزایش تاب آوری و خودکنترلی دارد (001/0<p).

    نتیجه گیری:

     این مطالعه نشان داد که آموزش مولفه های شناختی رفتاری و مولفه های مبتنی بر پذیرش و تعهد منجر به افزایش تاب آوری و خودکنترلی در زنان مطلقه شده و می توانند به عنوان روش های درمانی در بهبود مشکلات زنان مورد استفاده قرار گیرند.

    کلیدواژگان: درمان شناختی رفتاری، درمان پذیرش و تعهد، تاب آوری، خودکنترلی، زنان مطلقه
  • شاهو فاتحی، امید مرادی*، فایق یوسفی صفحات 509-523

    هدف :

    هدف از پژوهش حاضر اثربخشی آموزش مدل همسرآزاری بر کاهش گرایش به طلاق زنان مراجعه کننده به مراکز مشاوره شهر سنندج بود.

    روش پژوهش: 

    روش مطالعه حاضر جزء تحقیقات نیمه آزمایشی از نوع پیش آزمون- پس آزمون و پیگیری با گروه گواه محسوب می شود. جامعه آماری تحقیق شامل کلیه زنان مراجعه کننده به مراکز مشاوره طلاق، بهزیستی، اورژانس اجتماعی و مراکز مشاوره سطح شهر سنندج در سال 1400 بود که با استفاده از روش نمونه گیری در دسترس تعداد 400 نفر انتخاب شدند. از بین 400 نفر، تعداد 30 نفر از زنانی که در پرسشنامه گرایش به طلاق روزلت، جانسون و مورو (1986) نمره بالا را کسب کردند و حاضر به شرکت بودند به شیوه در دسترس انتخاب و در دو گروه آزمایش (15 نفر) و گواه (15 نفر) قرار گرفتند. پس از آموزش مدل بر اساس پروتکل آموزشی تدوین شده توسط محقق (گروه گواه هیچ آموزشی دریافت نکردند)، پس آزمون گرفته شد. داده ها با استفاده از آزمون تحلیل واریانس با اندازه گیری مکرر و نرم افزار SPSS-22 تحلیل شد.

    یافته ها

    نتایج نشان داد به دلیل اینکه سطح معناداری به دست آمده برای آزمون تحلیل واریانس با اندازه گیری مکرر کمتر از 05/0 می باشد، لذا اختلاف معناداری بین گرایش به طلاق زوجین در گروه آزمایش و گروه گواه وجود دارد و گرایش به طلاق زوجین (42/6=F، 017/0=P) در گروه آزمایش کمتر از گرایش به طلاق زوجین در گروه گواه می باشد.

    نتیجه گیری

    می توان نتیجه گرفت مدل علی همسرآزاری بر کاهش گرایش به طلاق زوجین اثر دارد. با آموزش همسرآزاری می توان رفتار همسران را نسبت زنان در جامعه تغییر و از بروز خشونت و همسرآزاری در روابط زناشویی جلوگیری نمود و در نهایت می توان از آسیب های بعدی ممانعت به عمل آورد.

    کلیدواژگان: گرایش به طلاق، همسرآزاری، زنان
  • فرح جعفری، محمدحسن غنی فر*، قاسم آهی صفحات 524-540

    هدف :

    هدف پژوهش حاضر، بررسی نقش ویژگی های شخصیت بر سلامت جسمانی با میانجی گری امید به زندگی و کنش های شناختی در زنان سالمند بود.

    روش پژوهش: 

    پژوهش حاضر از نوع توصیفی- همبستگی است و به روش مدل یابی معادلات ساختاری انجام شد. جامعه آماری شامل کلیه زنان سالمند مراجعه کننده به سراهای محله شهر تهران بین ماه های اردیبهشت تا خرداد ماه سال 1400 بود. نمونه پژوهش شامل تعداد 384 نفر از زنان سالمند مراجعه کننده به سراهای محله شهر تهران بود که با روش نمونه گیری داوطلبانه انتخاب شدند. داده ها با استفاده از پرسشنامه ویژگی های شخصیتی پنج عاملی نیو (مک کری و کاستا، 1980)، پرسشنامه توانایی های شناختی نجاتی (1392)، پرسشنامه امید به زندگی (هزاروسی، 1385) و پرسشنامه سلامت جسمی (اسپنس، هلمریچ و پرد، 1987) به دست آمد. داده ها با استفاده از ضریب همبستگی، ماتریس همبستگی پیرسون، رگرسیون چندگانه و مدل سازی معادلات ساختاری تحلیل شد. همچنین کلیه محاسبات آماری با استفاده از نرم افزار Lisrel 8.80 و SPSS.22 انجام گرفت.

    یافته ها

    نتایج نشان داد ضرایب استاندارد و غیر استاندارد مسیر مستقیم مدل فرضی از ویژگی-های شخصیت به کنش های شناختی (48/0= β؛ 01/0 P)، ویژگی های شخصیت به امید به زندگی (58/0= β؛ 01/0 P)، کنش های شناختی به سلامت جسمی (20/0= β؛ 01/0 P)، امید به زندگی به سلامت جسمی (30/0= β؛ 01/0 P) معنی دار شده است. بررسی شاخص های برازش مدل حاکی از آن است که مدل مورد نظر برازش مطلوبی با داده ها دارد. که امید به زندگی و کنش های شناختی اثر غیرمستقیم بر سلامت جسمانی داشته است، بنابراین وجود رابطه غیرمستقیم بین متغیرهای پژوهش با 95 درصد اطمینان تایید می شود (05/0>P).

    نتیجه گیری

    می توان نتیجه گرفت مدل ویژگی های شخصیت بر سلامت جسمانی با میانجی گری امید به زندگی و کنش های شناختی در زنان سالمند برازش دارد.

    کلیدواژگان: ویژگی های شخصیت، سلامت جسمانی، امید به زندگی، کنش های شناختی، سالمندی
  • فائزه ناصح جهقی، حسن میرزاحسینی*، نادر منیرپور صفحات 541-555
    هدف

    پژوهش حاضر با هدف تبیین سازمان شخصیت مرزی براساس روابط موضوعی اولیه، و مقاومت پذیری انجام شد.

    روش بررسی

    این پژوهش از نوع توصیفی بود که به روش همبستگی انجام شد. جامعه آماری شامل کلیه دانشجویان دوره کارشناسی و کارشناسی ارشد دانشگاه های تهران در سال تحصیلی 1400-1401 بود، که 412 نفر از آنها با استفاده از روش نمونه گیری در دسترس انتخاب شدند. شرکت کنندگان پرسش نامه های روابط موضوعی بل و همکاران (1985)، سازمان شخصیت مرزی کرنبرگ (2002) و مقیاس مقاومت پذیری عاشور پور و همکاران (2010) را تکمیل کردند. مدل یابی معادلات ساختاری با استفاده از داده های بدست آمده از پرسش نامه ها انجام شد.

    یافته ها: 

    نتایج نشان داد که روابط موضوعی به طور مستقیم 29/0 درصد و به واسطه مقاومت پذیری 19/0 درصد از ساختار شخصیت مرزی را تبیین می کند. در مجموع، برآورد مدل ساختاری که در آن روابط موضوعی متغیر پیش بین و مقاومت پذیری به عنوان متغیر میانجی وارد مدل شدند، نشان داد که این مدل 19/0 درصد از ساختار شخصیت مرزی را تبیین می کند (01/0>P).

    نتیجه گیری:

     بنابراین می توان چنین نتیجه گیری نمود که سازمان شخصیت مرزی تنها به صورت غیرمستقیم و از طریق مقاومت پذیری، توسط روابط موضوعی به صورت منفی تبیین می گردد.

    کلیدواژگان: روابط موضوعی اولیه، ساختار شخصیت مرزی، مقاومت پذیری
  • سلمان جوانبخت، زهرا یوسفی* صفحات 556-584
    هدف

    پژوهش حاضر با هدف بررسی روایی و پایایی مقیاس عملکرد اجرایی خانواده در بین افراد متاهل بالای 18 سال شهر اصفهان انجام شد. بنابراین پژوهش از نوع زمینه یابی و ابزارسازی بود.

    روش

    این پژوهش از نوع پیمایشی و ابزار سازی بود. جامعه آماری این پژوهش را کلیه زنان و مردان متاهل بالای 18 سال شهر اصفهان تشکیل می دادند. نمونه آماری400 مرد و زن بودند که به شکل در دسترس انتخاب شدند. ابزارهای پژوهش شامل مقیاس عملکرد اجرایی (جیوآ، اسکوید، گای و کنوردی،2000) و پرسشنامه تجربه هیجانی نسبت به همسر (غفرالهی، یوسفی، 1399) بود. داده ها در دو بخش توصیفی (میانگین و انحراف استادارد) و استنباطی (همبستگی پیرسون، آلفای کرانباخ و تحلیل عامل اکتشافی) تحلیل گردید.

    یافته ها: 

    نتایج نشان داد همسانی درونی (75/0=α)، پایایی بازآزمایی (450/0= r و 01/0>P) و هماهنگی سوالات مناسب بود. از آنجا که نمره بالاتر در عملکرد اجرایی خانواده نشان از عملکرد اجرایی ضعیف تر است شواهد بررسی روایی واگرا با مقیاس تجربه هیجانی منفی (576/0= r و 01/0>P) نسبت به همسر رابطه مثبت و معنادار نشان داد و نتایج روایی همگرا نشان داد بین نمره کل این مقیاس با مقیاس تجربه هیجانی مثبت نسبت به همسر (276/0- = r و 01/0>P) و عملکرد خانواده (711/0- = r و 01/0>P) رابطه منفی و معنادار وجود دارد همچنین بین عملکرد اجرایی با عملکرد اجرایی خانوادگی رابطه منفی و معنادار وجود داشت (671/0- = r و 01/0>P) که موید روایی همزمان است. نتایج تحلیل عامل اکتشافی نشان داد که این مقیاس از هشت عامل تشکیل شده است. همچنین همبستگی هر عامل با نمره کل معنادار بود که همگی موید روایی سازه این مقیاس هستند .

    نتیجه گیری:

     با توجه به این یافته ها می توان گفت که مقیاس عملکرد اجرایی از ویژگی های مطلوب روان سنجی برخوردار است و می توان از آن در حوزه آموزشی و پژوهشی و مشاوره استفاده کرد.

    کلیدواژگان: ویژگی های روانسنجی، عملکرد اجرایی، عملکرد اجرایی خانوادگی، عملکرد خانوادگی
  • فاطمه مسعودی همت آبادی، مهسا حاجی حسینی مسگر، مهرناز آزاد یکتا*، بیتا نصرالهی صفحات 585-601

    هدف :

    هدف از پژوهش حاضر بررسی رابطه تعارض کار- خانواده و استرس شغلی با خستگی عاطفی با میانجی گری توانمندی های منشی در کارمندان واحد بازرگانی شرکت هواپیمایی جمهوری اسلامی ایران (هما) بود.

    روش پژوهش: 

    پژوهش حاضر از نوع همبستگی است. به منظور نیل به هدف، از جامعه 864 نفری کارکنان، 265 نفر به روش نمونه گیری تصادفی ساده انتخاب شدند و پرسشنامه تعارض کار- خانواده کارلسون و همکاران (2000)، استرس شغلی هلریگل و اسلوکام (2000)، فرسودگی شغلی مسلش (1981) و توانمندی های منش سلیگمن و پترسون (2004) را تکمیل نمودند. داده های حاصله با استفاده از همبستگی پیرسون، تحلیل رگرسیون چندگانه، بوت استرپ و تحلیل مسیر و با بکارگیری نرم افزار SPSS22 مورد تجزیه و تحلیل قرار گرفت.

    یافته ها

    نتایج نشان داد که تعارض کار- خانواده و استرس شغلی به واسطه توانمندی های منشی با خستگی عاطفی رابطه داشته و این رابطه به صورت مستقیم و غیر مستقیم معنادار بود. بین تعارض کار- خانواده با خستگی عاطفی رابطه مثبت معنادار، بین استرس شغلی و خستگی عاطفی رابطه مثبت معنادار، بین تعارض کار- خانواده و استرس شغلی و توانمندی های منشی رابطه منفی معنادار، بین استرس شغلی و توانمندی منشی با خستگی عاطفی رابطه مثبت معنادار، بین توانمندی های منشی و خستگی عاطفی رابطه منفی معنادار وجود داشت (05/0>P).

    نتیجه گیری

    با توجه به ارتباط معنادار و منفی متغیر توانمندی های منشی با خستگی عاطفی و تعارض کار- خانواده، لازم است زمینه مناسب برای آشنایی کارکنان با مدیریت علمی تعارض کار- خانواده و آموزش و ارتقاء سطح این متغیر در کارکنان، از طرف سازمان ها مورد حمایت قرار گیرد.

    کلیدواژگان: تعارض کار- خانواده، استرس شغلی، خستگی عاطفی، توانمندی های منشی
  • اکرم مطهری نسب، کیومرث فرح بخش، احمد برجعلی، نورعلی فرخی صفحات 602-623

    هدف:

     با توجه به اهمیت روابط زوج در خانواده این پژوهش با هدف تدوین الگوی رضایت زناشویی در ایام کرونا اجرا شد.

    روش پژوهش:

     این پژوهش به صورت کیفی از نوع نظریه داده بنیاد صورت گرفته است. در این مرحله پژوهشگر به شناسایی مولفه‌های سازنده در رضایت زناشویی در ایام کرونا و مقایسه دو گروه دارای رضایت زناشویی و عدم رضایت زناشویی پرداخته است. در این مرحله روش نمونه گیری هدفمند و در دسترس بوده است. از بین این افراد 12 نفر از افرادی که دارای رضایت زناشویی پایین قرار داشتند و 12 نفر از افرادی که رضایت زناشویی خوبی داشتند، انتخاب شد و مصاحبه ای به عمل آمد.

    یافته‌ها:

    نتایج نشان داد عوامل اصلی موثر در رضایت زناشویی در ایام شیوع کرونا شامل هشت مورد عشق‌ورزی (همدلی، محبت، قدردانی، رضایت جنسی، درک متقابل و اهمیت دادن به خواسته های همسر)، مدیریت مرز (رعایت انصاف در تقسیم وظایف، مدیریت دخالت خانواده ها و حفظ استقلال، ایجاد حفظ حریم فرزندان و والدین، تامین نیازهای عاطفی والدین و حمایت و کمک به والدین)، تعادل کار خانوده (انعطاف‌پذیری در نقش‌ها، مسیولیت پذیری در انجام کارها، و اولویت دادن به خانواده در برابر کار بیرون)، دلبستگی ایمن (داشتن اعتماد به توانمندی های خود، داشتن دید مثبت نسبت به همسر و تعادل در صمیمیت و استقلال شخصی)، پختگی هیجانی (زنده نگهداشتن کودک درون، انعطاف‌پذیری و محدود کردن دخالت‌های خانواده‌ها)، اعتقادات مذهبی (اعتقاد به توکل به خدا، گذرا و موقتی بودن بودن درد و رنج، همکاری در خدمت رسانی خیرانه و حضور در مراسم مذهبی با رعایت پروتکل‌ها)، حل مساله (قناعت پیشه‌گی، گفت‌و‌گو کردن، رفع سوء تفاهم، مشورت کردن و کمک گرفتن از منابع بیرونی حمایتی) و مدیریت زمان (استفاده از فضای مجازی برای امور شخصی، تفریحات دو نفره بین زن و شوهر، داشتن اوقات فراغت برای خود و وقت گذاشتن برای تفریح خانواده) بود. 

  • سید علی سرمدی، بهنام قنبرپور *، جهانبخش غلامی صفحات 624-638

     هدف:

    هدف از پژوهش حاضر مقایسه تطبیقی اشتغال زوجه در نظام فقهی و حقوقی ایران و مصر و تاثیرآن بر سبک زندگی اسلامی بود.

    روش پژوهش: 

    پژوهش حاضر یک پژوهش مروری و توصیفی بود که با استفاده از منابع کتابخانه ای انجام شد.

    یافته ها

    در آموزه های مکتب اسلام، نه تنها حق کار زنان به رسمیت شناخته شده است، بلکه در کنار آن، استقلال مالی آنان و دستمزد عادلانه نیز مورد تاکید قرار گرفته است. در فقه زنان و سبک زندگی اسلامی، پیوند زناشویی حقوقی را برای هر یک از همسران به همراه می آورد که متناسب با جایگاه اجتماعی او است. در قبال این امتیاز از زن خواسته شده است که برای انجام برخی فعالیت ها و رفت و آمدها نظر همسر و اجازه وی را کسب کند. گستره مواردی که در آن ها زن موظف به کسب اجازه از شوهر است موضوعی است که میان مذاهب مختلف اسلامی و بلکه میان مجتهدان یک مذهب، آرا و فتاوای مختلفی را پدید آورده است. از نظر مبانی فقهی، مسیله اذن زوجه از زوج در آیات به صورت اشاره و در روایات به طور صریح مطرح شده است.

    نتیجه گیری

    نتایج نشان داد که در نظام حقوقی ایران و مصر، اشتغال زوجه یک حق مدنی محسوب شده است که تنها زمانی قابل سلب است که در تنافی با مصالح خانوادگی یا حیثیات زوج و زوجه بعد از اثبات در محاکم باشد. دیدگاه غالب در زمینه جایگاه اشتغال زوجه در حقوق ایران و مصر، این است که چنانچه زوجه حین ازدواج شاغل باشد و زوج بر این امر وقوف داشته باشد، در این صورت نیز حق مرد بر منع زوجه از اشتغال بر قوت خود باقی است، مگر آن که عقد نکاح مبنی بر ادامه اشتغال زوجه منعقد شده یا اشتغال ضمن عقد، شرط شده باشد، در این صورت این حق ساقط است

    کلیدواژگان: حق اشتغال، زوجه، سبک زندگی اسلامی، نظام حقوقی ایران، نظام حقوقی مصر
|
  • Shahrzad Shafaei, Bahram Mirzaian *, Ramezan Hasanzadeh Pages 1-17
    Aim

    The purpose of this study was to determine the effectiveness of cognitive rehabilitation on executive function and working memory function in the elderly with non-clinical depression.

    Methods

    The current research was an applied research and in terms of the research method, it was a semi-experimental type of pre-test-post-test and follow-up research with a control group. The statistical population of this research includes all elderly people suffering from non-clinical depression, 90 men and women who visited the Mehrgan counseling center between January 2020 and June 2021. The study sample was 30 elderly people who were selected by purposeful sampling and randomly divided into two equal intervention and control groups (15 people in each group). The follow-up period was conducted three months after the completion of the post-test. Cognitive rehabilitation treatment sessions were conducted in ten one-hour sessions weekly based on the protocol of Schulberg and Matir (2001). The data in this research was obtained by using the Beck II depression questionnaire (Beck, Steer and Brown, 1996), the executive function questionnaire, the working memory function questionnaire: by Valat Ezuvi, Pradat Dale and Ezuvi (2012). In this research, repeated measurement test and SPSS.22 software were used.

    Results

    The results showed that cognitive rehabilitation on executive function (F=49.86, P<0.001) and working memory function (F=31.53, P<0.001) in the elderly with non-clinical depression. It was effective and these results lasted until the follow-up period.

    Conclusion

    It can be concluded that cognitive rehabilitation is an effective method to improve executive function and working memory function in the elderly with non-clinical depression and it can be used to reduce the problems of the elderly.

    Keywords: Rehabilitation, executive function, Memory, Elderly, depression
  • Zahra Rezaei Rozbahani, Azita Chehri *, Gelavij Mahmoodi Pages 18-32
    Objective

    The aim of this study was to evaluate the effectiveness of acceptance and commitment therapy on sleep problems and problem solving.

    Methods

    This quasi-experimental study was a pre-test-post-test design with a control and follow-up group for 3 months. The statistical population of this study included all 14 to 16 year old female students studying in high school in Kermanshah in 1399-1400. By available sampling method, 60 female students were selected and randomly assigned to two equal groups of 20 people. Data were collected using a problem-solving questionnaire (Hepner and Patterson, 1988) and a sleep assessment questionnaire (Boyce et al., 1989). For data analysis, SPSS software version 25 was used with descriptive statistics, statistical assumptions and analysis of variance with repeated measures and Bonferroni test.

    Results

    The results showed that acceptance and commitment treatment was significantly effective in improving problem solving skills and its dimensions in the experimental group. Also, the results showed that acceptance and commitment therapy had a positive effect on problem solving skills of the experimental group.

    Conclusion

    Based on the findings of the present study, it can be concluded that acceptance and commitment therapy using techniques such as mental techniques. Self-observation as a context, cognitive fault techniques, values, and committed action can be used as an effective treatment to reduce sleep problems and improve problem-solving techniques in adolescent girls.

    Keywords: Acceptance, Commitment Therapy, sleep problems, problem solving, adolescents
  • Farahnaz Amirlou, Hayedeh Saberi *, Mitra Hakim Shoushtari Pages 33-53
    Aim

    The combination of mindfulness, parental self-efficacy and extra-parenting has a significant role in reducing parental stress, especially mothers, and parental stress management is important to promote well-being.

    Methods

    The aim of this study was to investigate the mediating role of parental mindfulness and self-efficacy in explaining the causal relationship between extravagance and parental stress in mothers of children with autism. Correlational research design based on structural equation modeling and statistical population including all mothers with children 2 to 14 years old with autism referred to medical clinics in Tehran in 1400 that 250 people were selected by purposive sampling and Abidin parenting stress questionnaires. (1995) answered the Mindfulness Questionnaire (Brown and Ryan, 2003), the Trans parenting Questionnaire (Hawke and Holden, 2006) and the Parental Self-Efficacy Questionnaire (Dumka et al., 1996). Data analysis was performed using AMOS software version 24.

    Results

    The results showed that the total path coefficient between extravagance and parental stress (p<0.01, β=-0.456) was negative and the path coefficient between mindfulness and parental stress (p <0.01, β=-0.474).) and parental self-efficacy and parental stress (p<0.01, β= -0.386) were negative and at the level of 0.01 were significant that parental self-efficacy and mindfulness were negatively related to the relationship between extra-parenting and parenting stress and mediated meaningfully.

    Conclusion

    Trans parenting directly and parental mindfulness and self-efficacy directly and indirectly predict parental stress in mothers of children with autism. In order to provide interventions to reduce or prevent high levels of parental stress in families, it is necessary to pay special attention to interventions based on mindfulness, self-efficacy and extravagance in parent-child relationships.

    Keywords: Parenting stress, Mindfulness, meta-parenting, parental self-efficacy, autism
  • Neghar Rousta, Majid Ghadami *, Hosein Keshavarz Afshar, Ghodsi Ahghar Pages 54-69
    Aim

    The effectiveness of group counseling was compared with Gestalt therapy and reality therapy on emotional regulation of adolescent girls in Tehran.

    Methods

    The current research was a semi-experimental type with a pre-test-post-test design with a control group and a follow-up phase. The statistical population included all teenage girls in the first year of high school in public schools in Tehran in the academic year 2018-2019. 45 people were selected using the purposeful sampling method and were randomly divided into three groups of 15 people. . The first experimental group received 10 90-minute sessions of group counseling with the Gestalt therapy approach, and the second experimental group received 10 90-minute sessions of reality therapy counseling. The research tool was the emotion regulation questionnaire of Garnevsky and Karij (2006). The data were analyzed using the method of variance analysis of repeated measures.

    Results

    The results showed that Gestalt therapy intervention (F = 5.88, P = 0.020) and Reality therapy intervention (F = 5.77, P = 0.023) were effective on the emotional regulation score, and this effect was stable in the follow-up phase.

    Conclusion

    Gestalt therapy group counseling and reality therapy have had a positive and significant effect on emotion regulation; therefore, group counseling, methodical Gestalt therapy and reality therapy are suitable methods to increase emotional regulation of teenagers. According to the obtained results, it is suggested that school counselors and psychotherapy centers use group counseling methods with Gestalt therapy and reality therapy to improve emotion regulation.

    Keywords: Gestalt therapy, Reality Therapy, emotion regulation
  • Seyedeh Fatemeh Bahari Sagalaksari, Mohammadkazem Fakhri *, Shaban Haydari Pages 70-89
    Aim

    The aim of this study was to determine the effectiveness of positive psychotherapy on body image satisfaction and happiness in adolescent girls dissatisfied with body image.

    Methods

    The present study was a quasi-experimental design with pre-test-post-test and follow-up design. Among the adolescent girls who were dissatisfied with the body image, a sample of 30 people was selected by purposive non-random sampling method and was randomly replaced in two groups of experimental (positive psychotherapy) and control. (15 people in each group). After evaluation, 30 adolescent girls who were dissatisfied with the body image, who scored lower than the cut score of the image satisfaction questionnaire and had the necessary conditions, were identified and then divided into two groups of 15, one experimental group and one control group. Then, 8 sessions of 90-minute positive psychotherapy of Seligman (2008) in the experimental group were performed in person and in accordance with health protocols. Data were collected using Happiness Questionnaire and Image Satisfaction Scale. Data were analyzed using repeated measures analysis of variance with SPSS-22 software.

    Results

    The results showed that in the post-test and follow-up stages, positive psychotherapy was effective on body image satisfaction (F=49.86, P<0.001) and happiness (F=13.53, P<0.001). And in the post-test and follow-up stages, treatment based on acceptance and commitment were effective on body image satisfaction and happiness, and in other cases no significant difference was observed.

    Conclusion

    In general, according to the results of the study, it is possible to use positive psychotherapy and therapy based on acceptance and commitment to improve the psychological problems of adolescent girls.

    Keywords: Positive psychotherapy, Happiness, Adolescent girls, body image
  • Zahra Akhgary, Hossein Ebrahimi Moghadam *, Rahim Davari Pages 90-115
    Aim

    The aim of this study was to investigate the presentation of a structural model for predicting distress tolerance based on dark personality traits and perceived social support mediated by defense styles in divorce-seeking clients.

    Method

    The research method was analytical-correlation by structural equation modeling. The statistical population of the present study was the clients seeking divorce in family counseling clinics in Tehran in 2021. The sample size of 260 people was selected by available sampling method and used by Simmons and Gahr (2005) Distress Tolerance Scale, Johnson & Webster's Dark Triangle (2010), Zimet et al.'s Multidimensional Perceived Social Support Scale (1988) and Defense Mechanism Questionnaire. Andrews et al. (1993) responded. The proposed model was then analyzed by structural equation modeling using SPSS and AMOS version 24.

    Results

    The results showed that direct paths of personality traits and defensive styles were significant on distress tolerance (P <0.01). The direct path of perceived social support on distress tolerance was not significant (P <0.01). Direct paths of personality traits on perceived social support and defensive styles (grown and immature) on distress tolerance were significant (P <0.01), but the coefficient of personality traits was not significant on disturbed defense style (01 / 0 <P). Also, the direct paths of perceived social support on defense styles (disturbed and immature) were significant (P <0.01), but the direct path coefficient of perceived social support on the developed defense style was not significant (0.01). 0 <P). The results also showed that the indirect effects of perceived social support through the mediating role of defense styles on distress tolerance were significant (RMSEA = 0.077, GFI = 0.912, IFI = 0.959 and CFI = 0.957), but the indirect path of personality traits on distress tolerance was not significant with the mediating role of defense styles.

    Keywords: Distress tolerance, Dark personality traits, perceived social support, defense styles, Divorce
  • Zahra Bagheri Masi Mordeh, Majid Zargham Hajebi *, Abbas Habib Zadeh Pages 116-134
    Aim

    The aim of this study was to determine the fit of structural equations of emotional breakdown based on identity confusion mediated by defense mechanisms.

    Methods

    The research method was descriptive and the correlation research design was structural equation modeling. The statistical population of this study included the statistical population of this study including all adolescent female students with emotional failure in the 2019-20 academic year of Nurabad city. The sample of this study included 250 people who were selected using purposive sampling method. Data were collected using the Love Trauma Syndrome Questionnaire (Ross, 1999), the Personality Organization Questionnaire (Kernberg, 2002) and the Defense Mechanisms Questionnaire (Andrews et al., 1993). Research data were analyzed using Pearson correlation coefficient, structural equation model and SPSS software version 22 and AMOS version 22. Significance level in this study was considered 0.05.

    Results

    The results showed that confusion had an indirect effect on emotional failure, so the existence of an indirect relationship between research variables is confirmed with 95% confidence (P <0.05).

    Conclusion

    It can be concluded that personality organization has a mediating role in the relationship between defense mechanisms and emotional failure.

    Keywords: personality organization, identity confusion, Emotional Breakdown, Defense Mechanisms
  • Nazanin Habibi Zanjani, Saeideh Barazian *, Hassan Ahadi Pages 135-149
    Aim

    To determine the effectiveness of emotional rational behavior therapy on body image, body mass index and reduce social anxiety in overweight adolescents.

    Methods

    The present research method was quasi-experimental with pre-test and post-test design and follow-up with a control group. The statistical population of this study consisted of adolescent girls and boys who had referred to clinics in Tehran in 2019 with the problem of overweight and obesity, from which 40 people were selected as a sample by available sampling method. After selecting individuals, they were randomly assigned to two experimental groups and one control group (20 people in each group). The experimental group intervention was group therapy based on emotional rational behavioral therapy (Ellis and Dryden, 1997) and consisted of eight sessions lasting 120 minutes. Data were analyzed using Littleton et al.'s (2005) Body Image Concern Questionnaire, Connor et al.'s (2000) Social Anxiety Inventory, and repeated measures analysis of variance using SPSS.22 software. Data were analyzed by repeated measures analysis of variance and SPSS.22 software. Significance level in this study was considered 0.05.

    Results

    The results showed that rational-emotional behavior therapy in body image (F=25.29, P<0.001), body mass index (F=3.59, P=0.036), and reduction of anxiety Social (F=5.49, P=0.026) Adolescents were effective overweight (P<0.001).

    Conclusion

    Based on the research findings, it can be said that rational-emotional behavior therapy can have positive effects on body image, body mass index and reduce adolescent social anxiety.

    Keywords: Emotional rational behavior therapy, body image, Body mass index, social anxiety
  • Ali Rajabi Vandechali, Ramezan Hasanzadeh *, Ghodratollah Abbasi Pages 150-168
    Aim

    The aim of this study was to determine the effectiveness of CBT on self-harming thoughts and psychological well-being in students with dissolved romantic relationships.

    Methods

    This was a quasi-experimental study with pre-test and post-test design and follow-up with two experimental and control groups. The statistical population consisted of 587 patients referred to the counseling center of Sari city, from which 60 students with dissolved romantic relationships were selected based on the inclusion and exclusion criteria. Participants completed the Sanson & Sanson (2010) Self-Damage Questionnaire and the Ryff Psychological Well-Being Questionnaire (1989) in three pre-test, post-test, and follow-up sessions. The experimental group received 8 sessions of Cognitive-behavioral therapy and the control group did not receive any intervention. Data were analyzed using repeated measures analysis method with SPSS.22 software.

    Results

    In the experimental group, the mean (standard deviation) of self-harming thoughts decreased from 18.53 (1.95) in the pre-test to 8.00 (1.64) in the post-test, but in the control group the mean (deviation) The criterion was self-injurious thoughts in the pre-test of 18.80 (2.07) and in the post-test of 18.93 (1.53), which did not differ. Also, in the experimental group, the mean (standard deviation) of psychological well-being in the pre-test was 43.60 (12.45) which increased to 97.13 (8.14) in the post-test, but in the control group, the average (standard deviation) of well-being Psychology was obtained in the pre-test of 40.80 (11.26) and in the post-test of 40.80 (11.30) which was not different.

    Conclusion

    The study showed that positive treatment reduced self-harming thoughts and suicidal thoughts and increased psychological well-being in students with dissolved romantic relationships.

    Keywords: cognitive-behavioral therapy, self-harming thoughts, Psychological Well-Being, dissolution of romantic relationships
  • Vahideh Alsadat Fatemi, Abdollah Shafie Abadi *, Javad Khalatbari, Abolhasan Farhangi Pages 169-184
    Aim

    The aim of this study was to determine the effectiveness of admission and commitment therapy on communication skills of female high school students.

    Methods

    The research method was quasi-experimental with a pre-test, post-test design with a control group with a 2-month follow-up phase. In this study, the statistical population was female high school students in Tehran in the academic year 2019-20. Using simple random sampling method, 30 of them were divided into two groups of experimental and control (15 people in each group). The experimental group underwent 12 90-minute sessions of acceptance and commitment therapy sessions adapted from Peterson, Efert, Fingold, and Davidson (2009); But the control group did not receive any intervention and remained on the waiting list. The Revised Communication Skills Questionnaire Queendom (2004) was used to collect information. The analysis of information obtained from the implementation of questionnaires was performed through SPSS software version 24 in two descriptive and inferential sections (analysis of covariance, one-way analysis of variance and Bonferroni post hoc test).

    Results

    The results showed that acceptance and commitment treatment in the post-test and follow-up stages compared to the control group had a significant effect on message perception (F=6.37, P=0.004), emotion regulation (F=8.62, P<0.001), listening (F=4.57, P=0.017), insight into communication (F=4.04, P<0.001), assertiveness (F=9.16, P<0.001) has teenage girls.

    Conclusion

    Based on the results of the present study, it can be said that acceptance and commitment therapy can be used as therapeutic methods to improve communication skills of adolescent girls in educational and medical settings.

    Keywords: Acceptance, Commitment Therapy, Communication skills, Students, girls
  • Effat Shirazi, Mehdi Ghasemi Motlagh *, Behrang Esmaeeli Shad, Abolfazl Bakhshipoor Pages 185-207
    Aim

    The aim of this study was to model borderline personality traits based on attachment: the mediating role of self-differentiation, and emotional dysregulation.

    Methods

    The present study is descriptive and correlational (using structural equation modeling). The statistical population studied in this study included all male and female undergraduate and graduate students of Tehran Azad University who were studying in the 2019-20 academic year. The sample of the present study included 370 people who were selected by purposive sampling. Data were obtained using the Borderline Personality Questionnaire (Leishnring, 1999), the Revised Relationships-Relationship Structure Questionnaire (Fraley, Valero Brennan, 2000), the Scorne and Friedlander Self-Differentiation Questionnaire (1998), and the Gretz and Roemer Emotion Regulation Difficulty Scale. (2004). In the present study, the mean, standard deviation, correlation and normality of the distribution of research variables were investigated using descriptive statistics. Also, in order to analyze the data and answer the research questions, the structural equation modeling method is used, which after confirmatory factor analysis - in the measurement model section, in the structural equation model section, the causal relationships between latent variables are examined. In the model part of the structural function, the intensity of causal relationships (direct, indirect and total) between latent variables and the amount of variance explained in the whole model is specified. SPSS and Amos software version 21 were used for this purpose.

    Results

    The results showed that attachment (β=0.41), self-differentiation (β=0.24), and emotion regulation (β=0.31) had a direct effect on borderline personality symptoms (P<0.001). Self-differentiation and emotional dysregulation mediated the relationship between borderline personality traits and attachment.

    Conclusion

    As a result, it can be concluded that the modeling of borderline personality traits based on attachment to the mediating role of self-differentiation, and emotional dysregulation has a favorable fit.

    Keywords: Borderline Personality, Attachment, Self-differentiation, Emotional dysregulation
  • Seyedeh Sara Mirzaian Gizehroud, Nader Monirpour *, MohammadEsmaeil Akbari, Majid Zargham Hajabi Pages 208-216
    Aim

    To investigate the structural model of quality of life in women with breast cancer based on type D personality with mediating the role of compliance.

    Research Method

    In this applied study, descriptive information of women with breast cancer referred to Shohada Tajrish Hospital and Azar Clinic in Tehran Collected in 1399-1399. The sample size was determined to be 440 people. Data collection tools included demographic quality of life questionnaire (EORTC QLQ-BR23), personality type scale D and general and specific compliance questionnaire of chronic patients by Hayes et al. (1994).

    Results

    The coefficient of variable trajectory of type D on the treatment adherence of women with breast cancer which is 0.694 and also the t-statistic is 7.121, the coefficient of trajectory of the disease on the quality of patients studied is 0.887 Also, a t-statistic of 9.283 and finally a variable coefficient of type D personality trait were observed on the quality of life of women with breast cancer (0.523) and also a t-statistic of 4.120, all of which had a significant effect.

    Conclusion

    Type D personality has a significant effect on the quality of life of women with breast cancer through mediation of adherence therapy and reduces the quality of life.

    Keywords: Quality of Life, Type D Personality, Adherence to treatment, breast cancer
  • Adel Amirkhanloo, Yarali Dousti *, Reza Donyavi Pages 217-242
    Aim

    To determine the effectiveness of emotional rational behavior therapy on body image, body mass index and reduce social anxiety in overweight adolescents.

    Method

    The method of the present study was quasi-experimental with pre-test and post-test design and follow-up with a control group. The statistical population of this study consisted of adolescent girls and boys who had referred to clinics in Tehran in 2019 with the problem of overweight and obesity, from which 60 people were selected as a sample by convenience sampling method. After selecting individuals, they were randomly assigned to two experimental groups and one control group (20 people in each group). The interventions of both experimental groups were group therapy and consisted of eight sessions of 120 minutes. Data were analyzed by repeated measures analysis of variance and SPSS.22 software. Significance level in this study was considered 0.05.

    Results

    The results showed that the two approaches of cognitive-behavioral and rational-emotional behavior therapy were effective in reducing body image, body mass index and reducing social anxiety of overweight adolescents (P<0.001).

    Conclusion

    Based on the research findings, it can be said that cognitive-behavioral approach and rational-emotional behavior therapy can have positive effects on reducing body image, body mass index and reducing social anxiety in adolescents.

    Keywords: cognitive-behavioral therapy, emotional intelligence training, social adjustment, Behavior Disorder
  • Sima Kamranifar, Zahra Eftekhar Saadi *, Farah Naderi, Sahar Safarzadeh Pages 243-260
    Aim

    The aim of this study was to compare the effectiveness of immunization training against stress and emotion regulation training on social adequacy and high-risk behaviors in Adolescent girls.

    Methods

    The present study was a quasi-experimental study with pre-test and post-test and follow-up with a control group. The statistical population of this study included all female high school students in Ramhormoz city in the academic year of 2019-20 who were studying in schools of this city. From this statistical population, 3 schools were selected by multi-stage random sampling method. Two schools were randomly assigned to experimental groups 1 and 2 and one school as a control group. The number of stress immunization training groups according to Mickenbaum (2007) protocol included 23 people and the emotion regulation training group (2002) included 22 people and the control group consisted of 25 people participated in this study. The instruments used in this study included the social adequacy questionnaire of Flanner et al. (1990), the risk-taking questionnaire of Iranian-born Mohammadzadeh and Ahmadabadi (2008). Data analysis was performed using repeated measures analysis of variance using SPSS.22 software.

    Results

    The results showed that there is a significant difference between the experimental groups and the control group in terms of social adequacy (F=34.64, P<0.001) and high-risk behaviors (F= 56.56, P<0.001). Also, it was found that emotion regulation training method has a greater effect on improving social adequacy and high-risk behaviors.

    Conclusion

    It can be concluded that stress immunization training and emotion regulation training were effective on social adequacy and high-risk behaviors in female students.

    Keywords: social adequacy, high-risk behaviors, Stress, emotion regulation
  • Avisa Khabiri Pooya, Mohsen Mohammadi *, Abolfazl Karami, Ishaq Rahimian Boogar Pages 261-283
    Aim

    The main purpose of this study is to develop a model of sexual satisfaction based on cognitive, social and moral factors, and to clarify the importance of these factors in improving the quality of sexual satisfaction of healthier people.

    Method

    This research is a structural equation in terms of applied purpose and descriptive method. The statistical population of this study included 300 people (129 men and 161 women) married and single who had sexual experience and ranged from diploma to doctorate. Data were obtained using Sexual Satisfaction Questionnaire, Adjustment Difficulties Questionnaire, Sexuality Questionnaire, Differentiation Scale Questionnaire, Interpersonal Trust Questionnaire and Moral Justice Questionnaire. All information obtained from this study was analyzed by SPSS software version 22 based on descriptive statistics such as frequency, mean and standard deviation and the path analysis method was used.

    Results

    The results showed that sexual satisfaction with shared sex according to the value of 0.427 with difficulty of emotional regulation to the value of 0.258, with differentiation to the value of 0.243, trust in interpersonal relationship to the value of 0.269, Moral justice in the amount of 0.175 is seen with sexual satisfaction.

    Conclusion

    Individual and interpersonal dimensions of human beings can affect sexual satisfaction. It is suggested that the necessary training be done according to different human dimensions in the form of a model that focuses on cognitive issues, interpersonal relationships and moral dimension and provide conditions for promoting sexual satisfaction and its continuation in a safe and healthy environment to continue. This route will help.

    Keywords: sexual satisfaction, Shared Sex, Emotional Difficulty Regulation, differentiation, Moral Justice
  • Soroush Shahbeik, Ameneh Moazedian *, Parviz Sabahi, Arsalan Khanmohammadi Otaghsara Pages 284-308
    Aim

    The aim of the present study was to investigate the depression prediction model based on early maladaptive schemas, resilience and perceived social support by considering the mediating role of self-compassion and hope in men and women with cancer.

    Methods

    The research method was descriptive correlation. The statistical population in this study included all cancer patients referred to Imam Khomeini and Milad hospitals in Tehran. The sample size was 300 patients were selected based on inclusion and exclusion criteria. Research instruments included the Nef Self-Sufficiency Questionnaire (2003), Young's early maladaptive Schemas Questionnaire (1998), Connor-Davidson Resilience Questionnaire (2005), Zimet et al.'s Perceived Social Support Questionnaire (1988), Beck depression Scale (1961) And Snyder (1991) Hope Questionnaire. The collected data were analyzed by structural equation modeling.

    Results

    The finding of data analysis showed that the depression prediction model based on initial maladaptive schemas, resilience and perceived social support, considering the mediating role of self-compassion and hope based on experimental data, has a good fit. Also, the direct and indirect effects of early maladaptive schemas, perceived resilience and social support were considered (p <0.05) regarding the mediating role of self-care and hope on the depression of cancer patients. Self-compassion directly affects the depression of cancer patients (p <0.05). Hope also directly affects the depression in cancer patients (p <0.05).

    Conclusion

    According to the research results, in order for cancer patients to function properly, on the one hand, it is necessary to have modified maladaptive schemas, on the other hand, their treatment t should be based on improving resilience and perceived social support.

    Keywords: depression, early maladaptive schemas, Resilience, perceived social support, self-sufficiency, hope
  • Zhoreh Moradi *, Forogh Jafary Pages 309-325
    Aim

    The aim of this study was to investigate the mediating role of latent communication aggression in the relationship between intolerance of indecision and extramarital relationships.

    Research Method

    This is a descriptive study of correlational research. The statistical population of the study consisted of couples who referred to counseling centers in Alborz province in 1398 for marital counseling. The research sample was selected by available sampling. The required sample size in this study was 260 people based on Cochran's method and 254 samples (including 127 males and 127 females) were selected taking into account the sample loss. Research instruments included the Whitley (2008) Attitudes Toward Extramarital Relationships Scale, the Freeston et al. (1994) Uncertainty Intolerance Scale (1994), and the Nelson & Carroll (2006) Hidden Communication Aggression Scale. In this research, path analysis method was used to analyze the data by observing the relevant assumptions.

    Results

    The results of data analysis showed that the model fit indices were in good condition. Intolerance of uncertainty directly and indirectly (considering the mediating role of latent communication aggression) affects extramarital relationships (p <0.05). Hidden aggression also directly affects extramarital affairs (p <0.05).

    Conclusion

    Based on the research findings, it can be said that correcting intolerance of uncertainty and latent aggression as preconditions of extramarital relationships, improves the quality of marital life of couples, and has a significant effect on preventing extramarital affairs.

    Keywords: Hidden communication aggression, intolerance of indecision, extramarital affairs
  • Malihe Afkhami Ardekani, Saeed Vaziri Yazdi *, Yasser Rezapour Mirsaleh, MohamadHosein Falah Pages 326-346

    One of the important issues in the field of compassionate parenting is the use of styles in accordance with the cultural and indigenous principles of Iranian society. Therefore, the aim of this research was to provide a native (Islamic-Iranian) model of compassionate parenting based on the opinion of experts. This research is of mixed type with the aim of validating the local model. The statistical population of this research in the qualitative and quantitative part will be all specialists in the field of parenting studies and native-Islamic patterns. In the qualitative part, the samples will be selected from among the experts in a targeted way and based on the theoretical saturation of the research experts, and in the quantitative part, in a non-random way. Is. Finally, the conclusion has been reached that the native Iranian Islamic parenting model is compassionate with God-centered components of parents' behavior, mother's conditions during pregnancy and breastfeeding, calling children to moral virtues, deep compassion for children, good prayers for parents and children, belief in Valuableness of children, proper education of children, correct and different upbringing in the first, second and third seven years, being a role model for parents' behavior, reaching the true self, and being a practical person have good quantitative validity and content validity. That is, it can be observed that all the factor loadings of the questions are higher than 0.5, and this indicates the confirmation of the construct validity of the items. Also, during the confirmatory factor analysis, it was also determined that the structural equation model in the present study has a good fit. Therefore, this model has sufficient validity.

    Keywords: Indigenous pattern, parenting, compassion, Parents
  • Mohamadreza Mehdigholi, Farideh Dokanei Fard *, Pantea Jahangir Pages 347-365

    The present study aimed to evaluate the fit of the prediction model of marital burnout based on differentiation, emotional intelligence and mental health mediated by resilience in couples. The research method was correlational, structural equation modeling type. The statistical population included all couples referring to family counseling centers in District 1 of Tehran in the second half of July to the end of September 2021, in which 248 couples (124 males and 124 females) were selected by non-random sampling method. They were evaluated using the Marital Burnout Scale (Pines, 1996), Emotional Intelligence (Shut et al., 1998), Resilience (Connor & Davidson, 2003), and Differentiation of Self Questionnaire (Skowron and Dandy, 2003) and General Health Questionnaire (Goldberg & Hiller, 1972) were evaluated. Findings obtained from the evaluation of the proposed model by structural equation using SPSS23 and Smart PLS3 software showed that the model fit indices have a desirable level. There is a significant relationship between differentiation (P< 0.05) and mental health (P<0.01) and marital burnout, but there is no significant relationship between emotional intelligence and marital burnout. Resilience also does not mediate the relationship between differentiation, emotional intelligence and mental health and marital burnout. Based on the findings of the present study, it can be said that differentiation and mental health play an important role in marital burnout of couples, so paying attention to these variables will help researchers and therapists in the field of couple and family in prevention and designing appropriate therapies.

    Keywords: Resilience, differentiation, Marital Burnout, mental health emotional intelligence
  • Roya Didevar, Elham Zarqam *, Morteza Andalib Kourayem Pages 366-387
    Aim

    Due to the lack of a Persian version of the present instrument, the present study examined the psychometric properties and functional structure of the Young Positive Schema Questionnaire in female students of the Islamic Azad University of Varamin.

    Methods

    The present study was descriptive-applied research with field method. Data collection tools included Young Lewis et al. (2018) positive schema questionnaire, the short form of Young Wilburne et al. (2002) schema questionnaire, Loveband Depression and Anxiety and Stress Scale (1995) and Young and Parenting Questionnaire. Et al. (2003). To evaluate the face and content validity of the opinion of 11 experts in the field of psychology and to evaluate the construct validity and internal reliability of 468 samples, for divergent validity of 135 samples and to evaluate the retest reliability of the data of 100 samples were analyzed. In this study, content validity method (CVR) and content validity index (CVI) were used to calculate content validity. In order to evaluate the internal reliability of the questionnaire, Cronbach's alpha method was used and in order to evaluate the time reliability of the retest method, divergent validity of Pearson correlation with SPSS software was used. Confirmatory factor analysis with LISREL software was used to evaluate the construct validity.

    Results

    The results showed that the 76-item model with 17 factors was approved. Pearson correlation coefficient between the scores obtained from the two performances ranged from 0.68 to 0.87 and all were significant (P<0.001) which indicates a high correlation between the two tests and as a result the reliability of the appropriate retest of all subscales of the Young positive schema questionnaire.

    Conclusion

    Therefore, it can be concluded that the 76-item model with 17 factors of Young Positive Schema Questionnaire is a desirable tool for identifying and evaluating positive schemas.

    Keywords: schema, early maladaptive schema, Validity, Reliability
  • Nasrin Rahbari Ghazani, Nader Hajloo *, Seyfollah Aghajani Pages 388-403
    Objective

    The aim of this study was to investigate the effectiveness of family empowerment based on socio-emotional learning on changing the emotional atmosphere of the family and prevention of addiction in adolescents with substance dependent parents.

    Materials and Methods

    The method of the present study was quasi-experimental with pre-test-post-test design and control and follow-up group. The statistical population consisted of adolescents with substance dependent parents who had referred to drug addiction treatment centers in Tabriz. 30 of them were sampled in a targeted and accessible manner and were randomly assigned to the experimental and control groups. Research data were collected using the Nordgah Fard Family Emotional Atmosphere Questionnaire (1994) and the Mousavi et al. Addiction Tendency Scale (2008) and analyzed by repeated measures analysis of variance.

    Results

    The results of the present study showed that family empowerment is effective on changing the emotional atmosphere of the family and prevention of addiction in adolescents with substance dependent parents and this effectiveness is stable in quarterly follow-up.

    Discussion and Conclusion

    According to the results of the present study, it can be concluded that family empowerment based on socio-emotional learning is one of the effective therapies on the emotional atmosphere of the family and addiction prevention.

    Keywords: Social-Emotional Learning, Family Emotional Atmosphere, Prevention, Addiction
  • Tayeb Ensandoost, AliAkbar Samari *, MohammadHossein Bayazi, Alireza Rajaei Pages 404-420
    Aims

    Pain as a general sense has biological bases, but the role of psychological factors in the process of pain perception is important. Therefore, the aim of this study was to determine the effectiveness of acceptance and commitment therapy on psychological well-being of patients with chronic musculoskeletal pain.

    Methods

    The method of the present study was quasi-experimental with pre-test-post-test design and follow-up with a control group. The statistical population of the study included all patients with chronic musculoskeletal pain in the summer of 2020 in Torbat-e-Jam, from which 30 eligible individuals were selected by convenience sampling and randomly divided into experimental and control groups (15 in each group). And answered the psychological well-being questionnaire. Were placed. Acceptance and commitment treatment was taught in eight sessions weekly. Statistical data were analyzed using repeated measures analysis of variance and SPSS-16 software at a significance level of α = 0.05.

    Findings

    The results showed that acceptance and commitment treatment significantly improved psychological well-being in the experimental group compared to the control group (P <0.05). Also, the effect of acceptance and commitment therapy remained constant until the follow-up stage.

    Conclusion

    Based on the findings of this study, it can be said that acceptance and commitment therapy was effective in increasing psychological well-being of patients with chronic musculoskeletal pain and this method can be used to improve psychological problems in patients with chronic musculoskeletal pain.

    Keywords: acceptance, commitment, Psychological Well-Being, Musculoskeletal
  • Afsaneh Khajevand Khoshli * Pages 421-436
    Objective

    The aim of this study was to evaluate the effectiveness of parenting education based on acceptance and commitment on parenting styles and anxiety in mothers with children with attention deficit / hyperactivity disorder.

    Method

    The quasi-experimental research design was pre-test-post-test with two experimental and control groups. The statistical population of this study was all mothers with children with attention deficit / hyperactivity disorder. The sample of this study included 30 mothers with children with attention deficit / hyperactivity disorder who were selected by convenience sampling method. Then, the mothers of the experimental group underwent 9 sessions of 90 minutes, one week of parenting training based on acceptance and commitment, but the control group did not receive any intervention. The research instruments were Beck Anxiety Questionnaire and Bamrind Parenting Style Questionnaire. Data were analyzed using repeated measures analysis of variance and Spss.23 software.

    Results

    The results showed that parenting education based on acceptance and commitment on parenting styles (authoritative (P<0.001)), authoritarian (P<0.001), negligent (P<0.001) and anxiety (P<0.001) had a significant effect on mothers with children with attention deficit / hyperactivity disorder.

    Conclusion

    It can be concluded that parenting education based on acceptance and commitment on parenting styles and anxiety in mothers with children with attention deficit / hyperactivity disorder can be effective and the results of this study can reduce the problems of mothers. Has a child with Attention Deficit / Hyperactivity Disorder.

    Keywords: Acceptance, Commitment Therapy, parenting styles, anxiety, Attention Deficit, Hyperactivity Disorder
  • Seyed Mohsen Ebrahimi, Rajabali Mohammadzadeh Admalaei *, Jamale Sadeghi, Aliasghar Abbasi Asfajir Pages 437-463

    The aim of this study was to investigate the mediating role of personality traits in the relationship between the intimacy of the source family and academic procrastination. To this end, 300 male and female students of Babylon Azad University completed the Family Origin Questionnaire (FOS), Solomon and Rothblum Academic Procrastination, and the Five Personality Traits Assessment (NEO-FFI). Data were analyzed using path analysis method through bootstrap test and structural equation modeling. According to the research findings, all personality traits of psychosis, extraversion, empiricism, agreement-seeking and conscientiousness could play a role as a mediating variable in relation to the family of origin and academic procrastination, but the characteristics of psychosis affect It was negative. In fact, it can be said that in a healthy family, the growth of intimacy and self-confidence will lead to the formation of personality traits such as activism, conscientiousness and empathy that affect academic achievement and reduce procrastination. While mentally disturbed people with a greater sense of insecurity and anxiety, usually with conflict and conflict, prevent the creation of an environment full of happiness and reduce the sensitivity of people to each other, also reduces the trust between them. They also hinder academic achievement and increase procrastination, instead of expressing emotions logically, with more aggressive and hostile behavior.

    Keywords: Origin family intimacy, personality traits, academic procrastination
  • Hosein Soleimanpour Moghaddam, Ali Saiedi *, Hossein Mahdian Pages 464-478

    The aim of the present study is comparison the effectiveness of compassion and acceptance and commitment rtraining (act) on psychological well-bing. This study was experimental with a pre-test and post-test design with control group. Research population consisted of 11th grade students in four districts of Mashhad in 2021-22 academic years. From schools that had at least 4 classrooms of 11th grade, one school were selected by cluster sampling. After visiting that school, three classes were randomly selected and divided lnto three equal groups.the experimental groups recieved 8 sessions of 90 minutes separately, with methods of the compassion and aecep tance and cammi tment - based training (act) and the control group didnot receive training.Research tools was pschological well-Bing scale (Ryff,1989).Data in spss-25 were analyzed with univaiate analysis of variance, with repeated measures and Bonferroni hoc methods. results indicated the effectiveness of both methods, compassion and acceptance and commitment training (act), in increasing psychological well-bing was (P˂0/001). Also was no significant difference between methods in increasing pschological well-Bing (P > 0/05). According to the findings of the present study,to increase pschological well-Bing for students,methods of compassion and acceptance and commitment training can be used.

    Keywords: compassion, Acceptance, commitment training, pschological well-Bing
  • Zohreh Sadeghi Amrabadi, Mahnaz Esteki *, Kambiz Pooshneh, Mahdieh Salehi Pages 479-492
    Aim

    This study aimed to determine the effectiveness of theraplay on symptoms of reactive attachment disorder in children aged 8-12 years.

    Methods

    Pre-test, post-test, and follow-up designs were used in this semi-experimental study. The statistical population included all Tehran youngsters aged 8 to 12 who had showed an interest in communicating via the internet. A total of 32 youngsters were chosen and assigned to one of two groups: experimental or control. The intervention was presented as theraplay weekly. Before and after the sessions, Randolph's attachment disorder questionnaire, Achenbach behavioral checker, and Wisconsin card classification test was completed and three months later the post-test was repeated. Data were analyzed using SPSS software using the mixed two-factor ANOVA method.

    Results

    The results of data analysis confirmed the significant effect of the theraplay intervention method on symptoms of reactive attachment disorder (P<0.001).

    Conclusion

    The usage of theraplay can be utilized as an effective treatment to alleviate the symptoms of reactive attachment disorder after a child's condition, according to the findings of this study.

    Keywords: Reactive attachment disorder, Theraplay, children
  • Hossein Ali Ghorbani Amir, Omid Moradi *, Mokhtar Arefi, Ahmadian Hamzeh Pages 493-508
    Aim

    The present study aimed to compare the effectiveness of cognitive behavior therapy (CBT) and Acceptance and Commitment Therapy (ACT) on resiliency and self-control strategies in divorced women.

    Methods

    The semi-experimental study was carried out on two experimental and control groups with a pre-test, post-test and follow-up design. The statistical population consisted of all divorced women referring to the Judicial Counseling Center of the Babol city in 2017-18. The sample of this study consisted of 45 women who were selected using the convenience sampling method and randomly divided into two experimental and control groups. Data collection tools included the Connor-Davidson Resilience Scale (CD-RISC) and Self-Control Questionnaire. Data were analyzed using the analysis of covariance test and SPSS.22 software.

    Results

    The study showed that CBT and ACT were effective on resiliency (P<0.001), and self-controlling (P<0.001) in divorced women. The study showed that acceptance and commitment therapy had a greater impact on increasing resilience and self-control than cognitive behavioral therapy (P<0.001).

    Conclusion

    The study showed that CBT and ACT led to an increase in resiliency and self-controlling in divorced women and can be used as treatment methods in improving women's problems.

    Keywords: Acceptance, Commitment Therapy, Cognitive Behavior Therapy, Resilience, self-control
  • Shaho Fatehi, Omid Moradi *, Fayeq Yousefi Pages 509-523
    Aim

    The aim of this study was the effectiveness of teaching the model of spousal abuse on reducing the tendency to divorce women referring to counseling centers in Sanandaj.

    Methods

    The method of the present study is a quasi-experimental research of pre-test-post-test and follow-up with a control group. The statistical population of the study included all women referring to divorce counseling centers, welfare, social emergency and counseling centers in Sanandaj in 2021, which were selected using the available sampling method of 400 people. Out of 400 people, 30 women who received a high score in the Roosevelt, Johnson and Moro (1986) Divorce Tendency Questionnaire and were willing to participate were selected by available means and divided into two experimental groups (15 people) and a control. (15 people) were included. After training the model based on the training protocol developed by the researcher (the control group did not receive any training), the post-test was performed. Data were analyzed using repeated measures analysis of variance and SPSS-22 software.

    Results

    The results showed that because the level of significance obtained for the analysis of variance with repeated measures is less than 0.05, so there is a significant difference between the tendency to divorce couples in the experimental group and the control group and the tendency Divorce of couples (F = 6.42, P = 0.017) in the experimental group is less than the tendency of divorced couples in the control group.

    Conclusion

    It can be concluded that the causal model of spousal abuse has an effect on reducing the tendency of couples to divorce. By teaching spousal abuse, the behavior of spouses towards women in society can be changed and the occurrence of violence and spousal abuse in marital relationships can be prevented, and finally, further harms can be prevented.

    Keywords: Tendency to divorce, spousal abuse, Women
  • Farah Jafari, MohammadHassan Ghanifar *, Qasem Ahi Pages 524-540
    Aim

    The aim of this study was to investigate the role of personality traits on physical health mediated by life expectancy and cognitive functions in elderly women.

    Method

    The present study is a descriptive-correlational study and was performed by structural equation modeling. The statistical population included all elderly women who referred to the neighborhoods of Tehran between May and June 2021. The research sample consisted of 384 elderly women referring to neighborhoods in Tehran who were selected by voluntary sampling method. Data were collected using the Neo Five Factor Personality Traits Questionnaire (McCurry and Costa, 1980), the Cognitive Rescue Abilities Questionnaire (2013), the Life Expectancy Questionnaire (Hazarousi, 2006) and the Physical Health Questionnaire (Spence, Helmrich). And Perd, 1987). Data were analyzed using correlation coefficient, Pearson correlation matrix, multiple regression and structural equation modeling. Also, all statistical calculations were performed using Lisrel 8.80 and SPSS.22 software.

    Results

    The results showed that standard and non-standard coefficients of the direct path of the hypothetical model from personality traits to cognitive actions (β = 0.48; P<0.01), personality traits to life expectancy (Β= 0.58; P<0.01), cognitive functions to physical health (β=0.20; P<0.01), life expectancy to physical health (β=0.30; P<0.01) has become meaningful. Examination of model fit indices indicates that the model has a good fit with the data. That life expectancy and cognitive functions have had an indirect effect on physical health, so the existence of an indirect relationship between research variables is confirmed with 95% confidence (P<0.05).

    Conclusion

    It can be concluded that the model of personality traits fits on physical health through the mediation of life expectancy and cognitive functions in older women.

    Keywords: personality traits, Physical Health, life expectancy, Cognitive Functions, aging
  • Faezeh Naseh Jahaghi, Hassan Mirzahoseini *, Nader Monirpoor Pages 541-555
    Objective

    The aim of this study was to explain the organization of border personality based on the initial thematic relationships and resilience.

    Methods

    This descriptive study was performed by correlation method. The statistical population included all undergraduate and graduate students of Tehran universities in the academic year 1401-1400, of which 412 were selected using the available sampling method. Participants completed the Bell et al. (1985) Thematic Relationships Questionnaire, the Kernberg Borderline Personality Organization (2002), and the Ashourpour et al. (2010) Resilience Scale. Structural equation modeling was performed using the data obtained from the questionnaires.

    Results

    The results showed that thematic relationships directly explain 0.29% and due to resilience 0.19% of the borderline personality structure. Overall, the estimation of the structural model in which the thematic relations of the predictor variable and resilience were entered as mediating variables, showed that this model explains 0.19% of the borderline personality structure (P<0.01).

    Conclusion

    Therefore, it can be concluded that borderline personality organization is only indirectly and negatively explained by thematic relationships through resistance.

    Keywords: Primary thematic relationships, Boundary personality structure, Resilience
  • Salman Javanbakht, Zahra Yousefi * Pages 556-584
    Objective

    The aim of this study was to investigate the validity and reliability of the Family Executive Performance Scale among married people over 18 years of age in Isfahan. The study consisted of all married men and women over 18 years of age in Isfahan. The statistical sample consisted of 400 men and women who were selected by convenience sampling. Data were analyzed in two parts: descriptive (mean and standard deviation) and inferential (Pearson correlation, Cronbach's alpha and exploratory factor analysis).

    Results

    The results showed internal consistency (0.75). = α), retest reliability (r = 0.450 and P <0.01) and coordination of questions were appropriate. Negative (r = 0.576 and P <0.01) showed a positive and significant relationship with the spouse and the results showed a convergent validity between the total score of this scale with the scale. Positive emotional experience towards spouse (r = -0.276 and P <0.01) and family performance (r = -0.711 and P <0.01) There is a significant negative relationship between executive performance and family executive performance. There was a negative and significant relationship (r = -0.671 and P <0.01) which confirms the simultaneous validity. The results of exploratory factor analysis showed that this scale consists of eight factors. Also, the correlation of each factor with the total score is significant.

    Conclusion

    According to these findings, it can be said that the Executive Performance Scale has desirable psychometric properties and can be used in the field of education, research and counseling.

    Keywords: Ecutive family function, Family Function, emotional experiences to spouse, Psychometric properties
  • Fatemeh Masoudi Hemmatabadi, Mahsa Haji Hosseini Mesgar, Mehrnaz Azad Yekta *, Bita Nasrollahi Pages 585-601
    Aim

    The aim of this study was to investigate the relationship between work-family conflict and job stress with emotional fatigue mediated by personnel capabilities in employees of the Commercial Unit of the Islamic Republic of Iran Airlines (Homa).

    Method

    The present study is a correlational study. In order to achieve the goal, from a population of 864 employees, 265 people were selected by simple random sampling method and Completed the Work-Conflict Questionnaire by Carlson et al. (2000), job stress by Hellerigel and Slocom (2000), Maslash burnout (1981) and the character skills of Seligman and Peterson (2004). The obtained data were analyzed using Pearson correlation, multiple regression analysis, bootstrap and path analysis using SPSS22 software.

    Results

    The results showed that work-family conflict and job stress were related to emotional fatigue due to secretory abilities and this relationship was directly and indirectly significant. There is a significant positive relationship between work-family conflict with emotional fatigue, a significant positive relationship between job stress and emotional fatigue, a significant positive relationship between work-family conflict and job stress and personnel capabilities, a significant positive relationship between job stress and secretarial ability with emotional fatigue, there was a significant negative relationship between secretory abilities and emotional fatigue (P<0.05).

    Conclusion

    Considering the significant and negative relationship between personnel capabilities and emotional fatigue and work-family conflict, it is necessary to provide a suitable background for employees to get acquainted with the scientific management of work-family conflict and training and promotion of this variable in employees by organizations.

    Keywords: work-family conflict, Job Stress, Emotional Fatigue, Personnel Capabilities
  • Akram Motahari Nasab, Kiumars Farah Bakhsh, Ahmad Borjali, Norali Farokhi Pages 602-623
    Aim

    Considering the importance of couples' relationships in the family, this study was conducted to develop a pattern of marital satisfaction during the corona period.

    Methods

    This qualitative study was conducted on the basis of data theory. In this stage, the researcher has identified the constructive components in marital satisfaction during the corona period and compared the two groups with marital satisfaction and marital dissatisfaction. Among them, 12 subjects with low marital satisfaction and 12 people who had good marital satisfaction were selected and interviewed. The codes were confirmed by 10 professors and finally comparisons were made between the two groups and a conceptual model was developed.

    Results

    The results of this study showed that the main factors affecting marital satisfaction during the coronavirus outbreak include eight cases of love (empathy, affection, appreciation, sexual satisfaction, mutual understanding and caring about the wishes of the spouse), border management (fairness in the division of duties, management of family involvement and independence, creating the privacy of children and parents, providing emotional needs of parents and supporting and assisting parents), family work balance (flexibility in role ( taking responsibility for things, and prioritizing the family against working outside), secure attachment (trusting in your abilities, having a positive view of your spouse and balancing intimacy and personal independence), emotional maturity (keeping the child alive, flexibility and limiting the involvement of families), religious beliefs (belief in trust in God, transient and temporary suffering, cooperation in the service of charity and good service). Attending religious ceremonies in accordance with protocols, problem solving (contentment, dialogue, resolving misunderstandings, consulting and getting help from external sources of support) and time management (using cyberspace for personal affairs, two-person recreation between husband and wife, having leisure time for yourself and taking time for family fun).

    Keywords: Corona outbreak, marital satisfaction, couple, qualitative study
  • Seyed Ali Sarmadi, Behnam Ghanbarpor *, Jahanbakhsh Gholami Pages 624-638
    Aim

    The aim of this research was to compare and contrast the dowry and alimony of women in the Islamic and Western legal systems.

    Method

    This study was conducted in a descriptive-analytical manner using library sources. Texts in the form of books and articles in Persian and English as well as newspapers were used in this way. In addition, reputable internet search engines such as Google and Yahoo were used in the development of this research.

    Results

    In this study, the financial rights of husband and wife towards each other, financial independence of couples, and laws related to dowry and alimony were examined in Islam and the West.

    Conclusion

    The Islamic legal system, according to the five schools of jurisprudence, has special financial privileges for the wife, which is almost unique compared to Western legal systems. One of these privileges is "dowry." Dowry is a mandatory gift that belongs to the wife and although it is not a condition for the validity of marriage, it becomes the responsibility of the husband as soon as the marriage contract is concluded, at least to the amount of the mahr al-mithl. Dowry is one of the financial rights of women in the Islamic family legal system. The judicial system of England and Wales is one of the most famous judicial systems in resolving divorce cases and observing equal rights for men and women, to the extent that London is called the divorce capital of the world. In the courts of this country, the man is considered as the main pillar of the family, but in case of divorce, the assets of both parties, and even their future retirement rights, will be examined and based on these examinations, excess assets beyond the needs of each party will be identified and divided equally between the wife and husband. This process is similar in most European and American countries, although structural differences in the law and judicial system of each country may lead to changes in the implementation of court orders

    Keywords: Comparison, contrast, dowry, alimony, Islamic legal system, West